Microbiology SBAs + EMQs Flashcards

1
Q

A 28 year old MSM presenting with painless, non-indurated penile ulcer. He also complains of rectal pain and bleeding, and on examination there is enlargement of the prostate. 1. Chlamydia trachomatis 2. Trichomonas vaginalis 3. Human Papillomavirus 4. Treponema Pallidium 5. Neisseria gonorrhoea 6. Candida albicans 7. Haemophilus ducreyi 8. Hepatitis C virus 9. HIV 10. Hepatitis B virus

A

1 - Chlamydia trachomatis. This is lymphogranuloma venereum, from serovars L1, L2 and L3.

How well did you know this?
1
Not at all
2
3
4
5
Perfectly
2
Q

A 24 year old woman presenting with vaginal discharge, on wet prep microscopy a flagellated pathogen is seen 1. Chlamydia trachomatis 2. Trichomonas vaginalis 3. Human Papillomavirus 4. Treponema Pallidium 5. Neisseria gonorrhoea 6. Candida albicans 7. Haemophilus ducreyi 8. Hepatitis C virus 9. HIV 10. Hepatitis B virus

A

2 - Trichomonas vaginalis. Flagellated protozoan, diagnosed with wet prep microscopy or PCR

How well did you know this?
1
Not at all
2
3
4
5
Perfectly
3
Q

A 29 year old man returns from travel abroad with several painful ulcers on his genitals. The pathogen is cultured on chocolate agar 1. Chlamydia trachomatis 2. Trichomonas vaginalis 3. Human Papillomavirus 4. Treponema Pallidium 5. Neisseria gonorrhoea 6. Candida albicans 7. Haemophilus ducreyi 8. Hepatitis C virus 9. HIV 10. Hepatitis B virus

A

7 - Haemophilus ducreyi Gram -ve coccobacillus (like Haemophilus influenzae), tropical ulcer, diagnosed on culture of chocolate agar

How well did you know this?
1
Not at all
2
3
4
5
Perfectly
4
Q

A person presents with positive VDRL and RPR tests, RPR titre falls after treatment with benzathine penicillin 1. Chlamydia trachomatis 2. Trichomonas vaginalis 3. Human Papillomavirus 4. Treponema Pallidium 5. Neisseria gonorrhoea 6. Candida albicans 7. Haemophilus ducreyi 8. Hepatitis C virus 9. HIV 10. Hepatitis B virus

A

4 - Treponema Pallidium (syphilis) Detect antibody with VDRL - can get biological false positives RPR is a modified VDRL test, more specific, titre falls in response to treatment so can use it to monitor the response.

How well did you know this?
1
Not at all
2
3
4
5
Perfectly
5
Q

A gram negative STI is picked up in a routine sexual health screen, using a NAAT (nucleic acid amplification test). 1. Chlamydia trachomatis 2. Trichomonas vaginalis 3. Human Papillomavirus 4. Treponema Pallidium 5. Neisseria gonorrhoea 6. Candida albicans 7. Haemophilus ducreyi 8. Hepatitis C virus 9. HIV 10. Hepatitis B virus

A

1 - Chlamydia trachomatis. Chlamydia is often asymptomatic, and cannot be cultured on agar, so diagnosed with a NAAT.

How well did you know this?
1
Not at all
2
3
4
5
Perfectly
6
Q

An alcoholic comes in to A&E with a cough productive of thick, purulent, red-stained sputum, and a fever. He is very unwell, and upper lobe changes are found on his chest x ray 1. Streptococcus pneumoniae 2. Staphylococcus aureus 3. Klebsiella pneumoniae 4. Moraxella catarrhalis 5. Legionella pneumophila 6. Mycobacterium tuberculosis 7. Pneumocystis jiroveci 8. Haemophilus influenza 9. Influenza 10. Mycoplasma pneumoniae

A

3 - Klebsiella pneumoniae Gm -ve rod-shaped bacillus - Cause atypical pneumonia, typically alcoholics - sudden, severe, systemic upset in these pts Production thick, purulent and sometimes blood-stained sputum = red-currant jelly Haemoptysis occurs more frequently than with other bacteria Radiological = upper lobe consolidation, marked cavitation

How well did you know this?
1
Not at all
2
3
4
5
Perfectly
7
Q

A man presents with sudden onset jaundice when he watched a football game in the cold. He is Coombs test positive, and also complains of a cough and joint pain, and is found to be anaemic 1. Streptococcus pneumoniae 2. Staphylococcus aureus 3. Klebsiella pneumoniae 4. Moraxella catarrhalis 5. Legionella pneumophila 6. Mycobacterium tuberculosis 7. Pneumocystis jiroveci 8. Haemophilus influenza 9. Influenza 10. Mycoplasma pneumoniae

A

10 - Mycoplasma pneumoniae Atypical pneumonia, organism has no cell wall. Commonly systemic symptoms, joint pain, erythema multiforme. One of the causes of COLD Autoimmune Haemolytic Anaemia

How well did you know this?
1
Not at all
2
3
4
5
Perfectly
8
Q

A woman who has had a renal transplant complains of a persistent fever, non-productive cough, weight loss and night sweats. Sputum investigation with silver stain reveals “boat shaped” cysts 1. Streptococcus pneumoniae 2. Staphylococcus aureus 3. Klebsiella pneumoniae 4. Moraxella catarrhalis 5. Legionella pneumophila 6. Mycobacterium tuberculosis 7. Pneumocystis jiroveci 8. Haemophilus influenza 9. Influenza 10. Mycoplasma pneumoniae

A

7 - Pneumocystis jiroveci PJP linked to immunosuppression Yeast-like fungus, fever, non-productive cough, wt loss, night sweats CXR = diffuse bilateral pulmonary infiltrates Dx = histological exam sputum, or BAL Gomori’s methenamine silver stain = “flying saucer”/”boat shaped” shaped cysts on microscopy

How well did you know this?
1
Not at all
2
3
4
5
Perfectly
9
Q

A middle aged man complains of a 2 week long “flu” with muscle aches and coryzal symptoms, now with a cough productive of sputum. Sputum microscopy shows b haemolytic gram positive cocci 1. Streptococcus pneumoniae 2. Staphylococcus aureus 3. Klebsiella pneumoniae 4. Moraxella catarrhalis 5. Legionella pneumophila 6. Mycobacterium tuberculosis 7. Pneumocystis jiroveci 8. Haemophilus influenza 9. Influenza 10. Mycoplasma pneumoniae

A

2 - Staph. Aureus Typically post-viral illness. B haem gm+ve, cocci, clusters, catalase +ve Consolidation, cavitation lungs, e

How well did you know this?
1
Not at all
2
3
4
5
Perfectly
10
Q

An elderly man with a long history of COPD and smoking presents with fever, chills, and pleuritic chest pain. A gram negative cocco bacilli is found on sputum microscopy 1. Streptococcus pneumoniae 2. Staphylococcus aureus 3. Klebsiella pneumoniae 4. Moraxella catarrhalis 5. Legionella pneumophila 6. Mycobacterium tuberculosis 7. Pneumocystis jiroveci 8. Haemophilus influenza 9. Influenza 10. Mycoplasma pneumoniae

A

8 - Haemophilus influenzae Associated with smoking, COPD Gram -ve cocco-bacilli

How well did you know this?
1
Not at all
2
3
4
5
Perfectly
11
Q

Treatment for young man presenting with neck stiffness, photophobia and fever, with true penicillin allergy 1. Cryptococcus neoformans 2. Neisseria meningitidis 3. Listeria monocytogenes 4. IM benpen 5. Streptococcus pneumoniae 6. Ceftriaxone 7. Coxsackie virus 8. Ceftriaxone + amoxicillin 9. Cefotaxime 10. Chloramphenicol

A

10 - Chloramphenicol

How well did you know this?
1
Not at all
2
3
4
5
Perfectly
12
Q

HIV +ve man presenting with meningism and focal neurology, on LP, high protein and high WCC with mononuclear cells 1. Cryptococcus neoformans 2. Neisseria meningitidis 3. Listeria monocytogenes 4. IM benpen 5. Streptococcus pneumoniae 6. Ceftriaxone 7. Coxsackie virus 8. Ceftriaxone + amoxicillin 9. Cefotaxime 10. Chloramphenicol

A

1 - Cryptococcus neoformans This CSF shows up in TB and cryptococcus infections

How well did you know this?
1
Not at all
2
3
4
5
Perfectly
13
Q

70 year old presenting with neck stiffness, photophobia and fever, 1st line treatment 1. Cryptococcus neoformans 2. Neisseria meningitidis 3. Listeria monocytogenes 4. IM benpen 5. Streptococcus pneumoniae 6. Ceftriaxone 7. Coxsackie virus 8. Ceftriaxone + amoxicillin 9. Cefotaxime 10. Chloramphenicol

A

8 - Ceftriaxone and amoxicillin >50 more susceptible to Listeria, E. Coli etc, need to add in amoxicillin.

How well did you know this?
1
Not at all
2
3
4
5
Perfectly
14
Q

2 month old presenting with irritability, neck stiffness, photophobia, and fever of no known source. This pathogen is not successfully treated with a B lactam alone. 1. Cryptococcus neoformans 2. Neisseria meningitidis 3. Listeria monocytogenes 4. IM benpen 5. Streptococcus pneumoniae 6. Ceftriaxone 7. Coxsackie virus 8. Ceftriaxone + amoxicillin 9. Cefotaxime 10. Chloramphenicol

A

3 - Listeria monocytogenes Neonates susceptible to Listeria, GBS, E.Coli etc, all of which need treatment with amoxicillin/ampicillin.

How well did you know this?
1
Not at all
2
3
4
5
Perfectly
15
Q

Teenager presenting to GP with neck stiffness, photophobia and fever, and non-blanching rash, immediate treatment 1. Cryptococcus neoformans 2. Neisseria meningitidis 3. Listeria monocytogenes 4. IM benpen 5. Streptococcus pneumoniae 6. Ceftriaxone 7. Coxsackie virus 8. Ceftriaxone + amoxicillin 9. Cefotaxime 10. Chloramphenicol

A

4 - IM Ben pen

How well did you know this?
1
Not at all
2
3
4
5
Perfectly
16
Q

A 23 year old woman returns from visiting her family in Russia and presents with several painless round black lesions surrounded by oedema. She says that she visited a petting zoo with her younger sister 1. Yersinia pestis 2. Bacillus anthracis 3. Leptospirosis interrogans 4. Bartonellosis 5. Influenza 6. Brucella melitensis 7. Leishmania tropica 8. Borrelia burgdorferi 9. Rabies 10. Coxiella burnettii

A

2 - Bacillus anthracis. This is cutaneous anthrax. Pulmonary anthrax (Woolsorters disease) is the deadly one

How well did you know this?
1
Not at all
2
3
4
5
Perfectly
17
Q

A 23 year old man competed in a triathlon 2 weeks ago and presents to A&E with high spiking temperatures, fatigue and muscle aches. He is found to have a gram negative, motile spirochaetes 1. Yersinia pestis 2. Bacillus anthracis 3. Leptospirosis interrogans 4. Bartonellosis 5. Influenza 6. Brucella melitensis 7. Leishmania tropica 8. Borrelia burgdorferi 9. Rabies 10. Coxiella burnettii

A

3 - Leptospirosis interrogans Leptospirosis/Weil’s disease. From water contaminated with dog/rat urine.

How well did you know this?
1
Not at all
2
3
4
5
Perfectly
18
Q

A 23 year old woman presents to the infectious diseases department with pyrexia of unknown origin. She has a long history of fluctuating fevers, arthralgia, myalgia, fatigue and lymphadenopathy. On taking a proper travel history, the ID department realises she has recently returned from the Middle East, where she milked goats 1. Yersinia pestis 2. Bacillus anthracis 3. Leptospirosis interrogans 4. Bartonellosis 5. Influenza 6. Brucella melitensis 7. Leishmania tropica 8. Borrelia burgdorferi 9. Rabies 10. Coxiella burnettii

A

6 - Brucella melitensis This is Brucellosis, get it from unpasteurised dairy products/meat/dairy animals. Classically called “undulating” fever (worse in evening, better in morning) Also has a funny smell (like wet hay)

How well did you know this?
1
Not at all
2
3
4
5
Perfectly
19
Q

A group of uni students return from hiking in the New Forest and present with flu-like symptoms and fever. On examination, they all have a rash, described in the notes as Erythema chronicum migrans. 1. Yersinia pestis 2. Bacillus anthracis 3. Leptospirosis interrogans 4. Bartonellosis 5. Influenza 6. Brucella melitensis 7. Leishmania tropica 8. Borrelia burgdorferi 9. Rabies 10. Coxiella burnettii

A

8 - Borrelia burgdorferi Lyme disease

How well did you know this?
1
Not at all
2
3
4
5
Perfectly
20
Q

A 25 year old man returns from travelling in South America with a crusty ulcer with raised edges on his leg. The pathogen is cultured on Novy-Macneal-Nicolle medium. 1. Yersinia pestis 2. Bacillus anthracis 3. Leptospirosis interrogans 4. Bartonellosis 5. Influenza 6. Brucella melitensis 7. Leishmania tropica 8. Borrelia burgdorferi 9. Rabies 10. Coxiella burnettii

A

7 - Leishmania tropica Cutaneous leishmaniasis

How well did you know this?
1
Not at all
2
3
4
5
Perfectly
21
Q

Anti-toxin vaccine against an upper-respiratory tract bacterial infection 1. Botulinum vaccine 2. Pneumococcal vaccine 3. MMR vaccine 4. Hepatitis A vaccine 5. Neisseria meningitidis vaccine 6. Tetanus vaccine 7. Diphtheria vaccine (single) 8. Rotavirus vaccine 9. Pertussis vaccine 10. Hepatitis B vaccine

A

7 - Diphtheria vaccine Caused by Corynebacterium diphtheriae, has both an anti-toxin and a toxoid vaccine (toxoid is given in pentavalent one)

How well did you know this?
1
Not at all
2
3
4
5
Perfectly
22
Q

Live attenuated vaccine that is given against a gastrointestinal pathogen, important in children

A

8 - Rotavirus

How well did you know this?
1
Not at all
2
3
4
5
Perfectly
23
Q

Vaccine given in a combination with others, against a pathogen that causes the “100 day cough” 1. Botulinum vaccine 2. Pneumococcal vaccine 3. MMR vaccine 4. Hepatitis A vaccine 5. Neisseria meningitidis vaccine 6. Tetanus vaccine 7. Diphtheria vaccine (single) 8. Rotavirus vaccine 9. Pertussis vaccine 10. Hepatitis B vaccine

A

9 - Pertussis vaccine

How well did you know this?
1
Not at all
2
3
4
5
Perfectly
24
Q

Toxoid vaccine used to treat bacterial infection that can cause umbilical cord stump infections 1. Botulinum vaccine 2. Pneumococcal vaccine 3. MMR vaccine 4. Hepatitis A vaccine 5. Neisseria meningitidis vaccine 6. Tetanus vaccine 7. Diphtheria vaccine (single) 8. Rotavirus vaccine 9. Pertussis vaccine 10. Hepatitis B vaccine

A

6 -Tetanus vaccine Sorry, this is a bit mean. In developing countries neonatal tetanus is a big thing, it causes umbilical cord stump infections

How well did you know this?
1
Not at all
2
3
4
5
Perfectly
25
Q

Live attenuated vaccine given to prevent extremely contagious coryzal exanthema caused by a morbillivirus 1. Botulinum vaccine 2. Pneumococcal vaccine 3. MMR vaccine 4. Hepatitis A vaccine 5. Neisseria meningitidis vaccine 6. Tetanus vaccine 7. Diphtheria vaccine (single) 8. Rotavirus vaccine 9. Pertussis vaccine 10. Hepatitis B vaccine

A

3 - MMR vaccine Measles - very contagious exanthema, gives you coryzal symptoms. Caused by morbillivirus.

How well did you know this?
1
Not at all
2
3
4
5
Perfectly
26
Q

A 24 year-old Asian man presents with a persistent cough. A sputum sample is taken and cultured on Lowenstein–Jensen medium, appearing as brown, granular colonies after several weeks. The organism implicated is: A Coxiella burnetti B Streptococcus pneumoniae C Mycobacterium tuberculosis D Legionella pneumophilia E Mycobacterium leprae

A

C Mycobacterium tuberculosis This gentleman is most likely suffering from mycobacterium tuberculosis which characteristically presents with a persistent cough, haemoptysis, fever, night sweats and weight loss. Lowenstein–Jensen medium is a growth medium used to culture Mycobacterium species at 37°C. The most common indication for its use is to culture Mycobacterium tuberculosis (C), where it appears as brown coffee-coloured (buff), granular bread crumb-like colonies (rough) which often stick to the bottom of the growth plate and are hard to remove (tough). This is often remembered as ‘buff, rough and tough’. It usually takes approximately 4–6 weeks to obtain these visible colonies, an important fact to remember when treating patients. Another characteristic feature is the formation of serpentine rods from chains of cells in smears. There are a few other important points to remember about staining results for Mycobacterium tuberculosis. They are classified as acid-fast bacteria, because they are resistant to losing their colour during staining procedures. The Ziehl–Neelson stain is the most common method used to stain this type of bacterium, and they appear bright red against a blue background. The stain contains carbofuchsin, a pink dye which binds to the unique mycolic acids found in the mycobacterium cell wall. Another stain that can be used for acid-fast bacilli is the auramine stain, which also binds to mycolic acids to give a yellow fluorescence.

How well did you know this?
1
Not at all
2
3
4
5
Perfectly
27
Q

A 24-year-old HIV-positive Asian man presents with a cough. A Mantoux test is performed. After 72 hours, the wheal diameter is measured at 5.8 mm. This indicates: A He has never been exposed to TB B He has been exposed to TB C He has had a BCG vaccination in the past D He has latent TB which is now reactivated E It is not possible to say

A

B He has been exposed to TB The Mantoux test is a diagnostic test for tuberculosis. It consists of an intradermal injection of 0.1 mL of purified protein derivative (PPD) tuberculin, which is a glycerol extract of the bacillus. The diameter of the induration that subsequently forms is read 48–72 hours later, but one also needs to take into account the patient’s risk of being infected with TB and of progression to disease if they were infected in interpreting the result. The Centers for Disease Control and Prevention provide the following classification for the skin test: 1 An induration of 5 mm or more is considered positive in: • Patients with HIV • A recent contact of a person with TB disease • People with fibrotic changes on chest radiograph consistent with prior TB • Patients with organ transplants • People who are immunosuppressed for other reasons (for example taking the equivalent of >15 mg/day of prednisone for 1 month or longer) 2 An induration of 10 mm or more is considered positive in: • Recent immigrants (

How well did you know this?
1
Not at all
2
3
4
5
Perfectly
28
Q

An 18-year-old university student develops a lower lobe pneumonia, with a raised white cell count and CRP. A sputum culture reveals a Gram-positive optochin-sensitive diplococcus. The most likely causative agent is: A Staphylococcus aureus B Streptococcus viridans C Mycoplasma pneumoniae D Streptococcus pneumoniae E Haemophilus influenzae

A

D Streptococcus pneumoniae It is useful to remember that streptococci can essentially be divided into alpha haemolytic, beta haemolytic and non-haemolytic groups. Alpha haemolytic streptococci can be further divided into Strep. pneumoniae (D) and Strep. viridans (B) according to their optochin sensitivity (amongst other factors). The beta haemolytic streptococci are further classified according to Lancefield groups A, B, C, F and G. Finally the non-haemolytic streptococci include the enterococci. Optochin is an antibiotic used to differentiate Strep. pneumoniae from other alpha haemolytic streptococci such as Strep. viridans. The pneumococcus will typically produce a zone of inhibition around an optochin disc, indicating that it is sensitive to the antibiotic, whereas Strep. viridans is resistant to it so its growth will not be affected. This can be remembered using the mnemonic ‘OVeR PS’ (Optochin – Viridans Resistant, Pneumococci Sensitive). As the organism in the question is optochin sensitive, the answer is (D).

How well did you know this?
1
Not at all
2
3
4
5
Perfectly
29
Q

A 58-year-old Caucasian alcoholic man presents to his GP with a history of sudden onset high fever, flu-like symptoms and, thick, blood stained sputum. Achest x-ray is arranged which shows marked upper lobe cavitation. The most likely causative agent is: A Klebsiella pneumoniae B Mycobacterium tuberculosis C Staphylococcus aureus D Moraxella catarrhalis E Pnemocystis jirovecii

A

A Klebsiella pneumoniae Klebsiella pneumoniae (A) is a Gram-negative rod-shaped bacillus that can cause an atypical pneumonia, most frequently in alcoholics. It can result in sudden, severe systemic upset in these patients, and the production of thick, purulent and sometimes blood-stained sputum said to resemble ‘red-currant jelly’. Haemoptysis occurs more frequently with K. pneumoniae than with pneumonia caused by other bacteria. Radiological features can include upper lobe consolidation, with marked cavitation as described in the question. It is more likely to lead to complications such as lung abscesses and empyemas than pneumonias caused by Strep. pneumoniae.

How well did you know this?
1
Not at all
2
3
4
5
Perfectly
30
Q

A 27-year-old intravenous drug user presents with a 2-week history of fevers, weight loss and a systolic murmur. The most likely causative agent is: A Streptococcus viridans B Candida albicans C Staphylococcus aureus D Streptococcus bovis E Kingella

A

C Staphylococcus aureus Infective endocarditis can be classified into two broad categories: acute and sub-acute. Acute infective endocarditis is less common, and the most likely causative agent is Staphylococcus aureus (C). It can affect both normal and abnormal valves, and can typically be found in intravenous drug users, such as the patient described. The tricuspid valve is most commonly affected in these cases, which can easily be remembered as this is the first valve that the bacteria will encounter following injection into a vein. Therefore, (C) is the correct answer in this case.

How well did you know this?
1
Not at all
2
3
4
5
Perfectly
31
Q

A patient with shingles is treated with an anti-viral. The drug used is a guanosine analogue and acts as a substrate for viral thymidine kinase. The most likely drug she has been given is: A Foscarnet B Lamivudine C Cidofovir D Acyclovir E Ganciclovir

A

D Acyclovir Acyclovir (D) is a guanosine analogue that causes obligate chain termination when it attaches to DNA. It is phosphorylated by the enzyme thymidine kinase found in viruses, which is far more effective than the cellular thymidine kinase for this process. This means that normal cells which are not infected by the virus are not affected as much by acyclovir, as there is no viral thymidine kinase present. The acyclovir monophosphate which then forms is further phosphorylated to a diphosphate and then to a triphosphate by the cellular thymidine kinase. This triphosphate potently inhibits viral DNA polymerase, leading to chain termination. It is effective against the herpes viruses, for example herpes simplex and herpes zoster which causes shingles.

How well did you know this?
1
Not at all
2
3
4
5
Perfectly
32
Q

According to the UK immunization schedule, which vaccine should be given to a 2-month-old baby who has already received DTaP (diptheria, tetanus, pertussis), IPV (polio) and Hib (haemophilus influenzae type B) vaccines? A Pneumococcus B MMR C Meningitis C D BCG E Hepatitis B

A

A Pneumococcus The current UK immunization schedule is as follows: • Two months: Hib/IPV/DTaP/PCV • Three months: Hib/IPV/DTaP/Men C • Four months: Hib/IPV/DTaP/PCV/Men C • Twelve months: Hib/Men C • Thirteen months: MMR/PCV • Three years four months old or soon after: MMR/DTaP/IPV • 13–18 years: Booster Diptheria and tetanus/IPV

How well did you know this?
1
Not at all
2
3
4
5
Perfectly
33
Q

A 24-year-old sexually active woman presents to her GP with dysuria. A urinary tract infection is diagnosed. Which of the following is the most likely causative agent? A Enterobacter B Escherichia coli C Klebsiella pneumoniae D Staphylococcus saphrophyticus E Proteus mirabilis

A

B Escherichia coli The most common cause of a urinary tract infection in all groups of patients is Escherichia coli (B). Do not be misled by the fact that the patient is a young, sexually active woman. The E. coli bacterium is a lactose-fermenting Gram-negative rod. It has various properties that aid its pathogenesis: a flagellum to enable it to move upstream, fimbrae so that it can adhere to the urothelium, and haemolysin to form pores in white blood cells. It also has a protective capsule called the K-antigen. The other lactose fermenting organisms are Klebsiella and Enterobacter, whilst non-lactose fermenting organisms include Proteus and Pseudomonas. Lactose fermenting organisms turn MacConkey agar pink, whereas non-lactose fermenters do not. Useful investigations for urinary tract infections can include a urine dipstick to look for nitrites and leukocytes, and urine cultures looking for a bactiuria of greater than 105 colony forming units

How well did you know this?
1
Not at all
2
3
4
5
Perfectly
34
Q

A 44-year-old woman patient returns from her holiday in India with a 2-day history of watery, offensive diarrhoea, bloating, excessive flatulence and abdominal pain. The GP obtains a stool sample. Microscopy reveals a flagellate pearshaped protozoan. The most likely organism implicated is: A Bacillus cereus B Salmonella enteritidis C Giardia lamblia D Entamoeba histolytica E Cryptosporidium parvum

A

C Giardia lamblia Giardia lamblia (C) is a flagellated protozoan parasite which causes giardiasis. It attaches to the small bowel wall, but does not invade it. If you can remember this fact, you will find it easier to remember that it interferes with absorption, and so leads to the classic symptoms of weight loss, flatulence, chronic diarrhoea and bloating, as in the patient in this question. Because it does not invade the small bowel wall, the diarrhoea is not bloody but it is watery. Microscopy of a stool sample may show a pear-shaped protozoan. If you imagine a pear making you feel very bloated, you will remember this fact which often crops up in questions! Very rarely, a string test may be done if other methods to detect the parasites fail but there is still a high index of clinical suspicion. A gelatine capsule attached to a long string is swallowed, with the end of the string remaining outside the mouth and taped to the patient’s cheek. It remains in place for about 4–6 hours, before the end is examined under the microscope. Treatment of giardiasis is typically oral metronidazole.

How well did you know this?
1
Not at all
2
3
4
5
Perfectly
35
Q

A 21-year-old medical student returns from her elective in India with a history of abdominal cramps, vomiting, fevers and profuse, watery stools which she describes as resembling ‘rice-water’. The GP obtains a stool sample. Analysis reveals curved, comma shaped organisms that were shown to be oxidase positive. The most likely organism implicated is: A Hepatitis A B Clostridium difficile C Yersinia enterocolitica D Campylobacter jejuni E Vibrio cholerae

A

E Vibrio cholerae Vibrio cholerae (E) causes profuse watery diarrhoea and vomiting. It can in fact be one of the most rapidly fatal infectious illnesses if not treated, because of the severe dehydration causing circulatory shock. The bacteria produce a toxin which has an A and a B subunit. It is the A subunit which activates a G protein and results in the production of cAMP, which initiates the secretion of Na+, K+, Cl-, and HCO3 - into the small intestine lumen. Most people only have a mild illness which simply resembles other diarrhoeal illnesses. Sometimes, as in this case, the diarrhoea is profuse and is known colloquially as ‘rice-water’ stools because of its appearance. The diagnosis is predominantly clinical, but if stool culture is performed the classical appearance will be of curved shaped, oxidase-positive organisms. You can remember this as the Cholera Comma! Rehydration therapy forms the mainstay of treatment.

How well did you know this?
1
Not at all
2
3
4
5
Perfectly
36
Q

A 35-year-old HIV-positive man presents to his GP complaining of a general feeling of tiredness, weight loss and night sweats. On examination there is hepatosplenomegaly and hyperpigmentation of the skin. The most likely diagnosis is: A Visceral leishmaniasis B Cutaneous leishmaniasis C Mucocutaneous leishmaniasis D Malaria E Schistosomiasis

A

A Visceral leishmaniasis Leishmaniasis is transmitted by phlebotomine sandflies and occurs in Africa, America and the Middle East. Visceral leishmaniasis (A) is also known as ‘Kala-azar’, and the most common clinical features include fever and splenomegaly. Hepatomegaly, skin hyperpigmentation and dry warty skin occur less frequently, and bone marrow invasion can result in pancytopenia. It can be mistaken for malaria, which is dangerous as it can be fatal if left untreated. L. donovani and L. infantum are thought to cause the disease in Africa, Asia and Europe, whilst L. chagasi is implicated in South America.

How well did you know this?
1
Not at all
2
3
4
5
Perfectly
37
Q

A 22-year-old student presents to accident and emergency with a raised, erythematous, scaly ulcer on his forearm which has not been healing. On examination he is also found to have lymphadenopathy. He gives a history of recently returning from a 2-month trek in the rainforests of South America. Tissue is aspirated from the margin of the ulcer, and the organism is cultured in Novy–MacNeal– Nicolle medium. The organism implicated is: A Toxoplasma gondii B Treponema pallidum C Leishmania dovani D Leishmania major E Leishmania braziliensis

A

D Leishmania major The picture described is consistent with cutaneous leishmaniasis, the most common form of leishmaniasis. An itchy, scaly papule develops at the bite site and develops into a crusty ulcer with raised edges. Local lymphadenopathy can also occur, but the lesion usually heals within 8 months leaving a depigmented scar called an oriental sore. The organisms implicated are Leishmania major (D) and L. tropica. You can remember this if you picture lots of skin lesions cropping up in travellers from the ‘major tropics’! It is found in many countries, ranging from South America to the Middle East. Diagnosis can be by Giemsa staining of slit skin smears, or from tissue aspirated from the ulcer. The organism can be cultured on Novy–Macneal–Nicolle medium as described in the question.

How well did you know this?
1
Not at all
2
3
4
5
Perfectly
38
Q

A 35 year-old male clothing merchant has returned to the UK 2 weeks ago from a visit home to Syria. A week later he presents with flu-like symptoms, drenching sweats and a recurring fever and is beginning to complain of lower back pain. After further questioning, he mentioned that he worked on a farm during his trip. He is successfully treated with oral doxycycline and gentamicin. What is the most likely diagnosis? A Malaria B Tuberculosis C Influenza D Brucellosis E Typhoid

A

D Brucellosis The Brucella species are Gram-negative, rod shaped, intracellular bacteria that cause a highly contagious zoonosis known as brucellosis (D). The causative agent in cattle is B. abortis, but in dogs it is B. canis. Infection in cattle can lead to miscarriages, hence the name ‘abortis’. Infection is usually contracted from unsterilized milk, cheese or meat. Clinical features of brucellosis can include a long history of undulating fevers, arthralgia and myalgia, weight loss, fatigue, lymphadenopathy, sacroilitis and depression. Many cases present as pyrexia of unknown origin. Hepatomegaly and/or splenomegaly can sometimes be found on examination.

How well did you know this?
1
Not at all
2
3
4
5
Perfectly
39
Q

A 50-year-old man has returned from hiking a segment of the Appalachian Trail on the Eastern coast of the USA during the summer months. Ten days later he presents to casualty with flu-like illness and a rash showing some central fading. What is the most likely organism implicated? A Herpes simplex B Epstein–Barr virus C Streptococcus pyogenes D Treponema pallidum E Borrelia burgdorferi

A

E Borrelia burgdorferi Borrelia burgdorferi (E) is a Gram-negative bacterium that causes Lyme disease. It is a spirochaete, which is the name for a group of bacteria that are helically coiled in shape. Lyme disease is actually thought to be the most common vector borne disease in England and Wales. It is named after a town called Lyme in Connecticut, where the disease was first seen. The vector is a tick called the Ixodes tick, which can be found on deer and rodents. Lyme disease is a multisystemic disorder which has three main stages: the local stage, disseminated stage and a late stage. The local stage involves a characteristic skin lesion called erythema chronicum migrans, usually appearing 7–10 days after the initial infection. It usually starts off as a red macule or papule, and approximately 1 week later expands to leave a target appearance with an area of central fading. Other symptoms at this stage are usually constitutional, such as a fever and headache. The somewhat unusual features of the next stage can be remembered using the word PEACH: Peripheral neuropathy, Erythema chronicum migrans (persists in this stage), Arthritis, Cranial nerve palsies and Heart block. Finally, the late stage can include persistent arthritis and chronic encephalitis. Treatment is with oral antibiotics, usually doxycycline.

How well did you know this?
1
Not at all
2
3
4
5
Perfectly
40
Q

A 26-year-old squash player is admitted with a red, swollen left knee. He reports no history of trauma. On examination he has a temperature of 38°C. A joint aspirate is taken. What is the most likely causative organism? A Neisseria gonorrhoeae B Staphyloccocus aureus C Haemophilus influenzae D Streptococcus viridans E Chlamydia trachomatis

A

A Neisseria gonorrhoeae The most common cause of septic arthritis in young, sexually active adults is Neisseria gonorrhoeae (A). A Gram-stain of this aspirate would reveal Gram-negative diplococci. It is less likely for this organism to lead to joint destruction than a staphylococcal arthritis. The two forms of disseminated gonoccocal infection are the septic arthritis form (as described in this case), and the bacteraemic form. Other clinical features of the bacteraemic form might include a migratory polyarthralgia and a vesicular or papular rash.

How well did you know this?
1
Not at all
2
3
4
5
Perfectly
41
Q

A 26-year-old squash player is admitted with a red, swollen left knee. He reports no history of trauma. On examination he has a temperature of 38°C. A joint aspirate is taken which grows Gram-negative diplococci. What is the antibiotic treatment regimen of choice for this patient? A Oral flucloxacillin for 4–6 weeks B IV flucloxacillin for 4–6 weeks C IV flucloxacillin for 2–4 weeks D IV flucloxacillin and vancomycin for 6–8 weeks E IV cefotaxime for 4–6 weeks

A

E IV cefotaxime for 4–6 weeks The patient in this question is presenting with septic arthritis, and the most likely cause given the joint aspiration findings of Gram-negative diplococci is Neisseria gonorrhoeae. The British National Formulary (BNF) advises the use of intravenous cefotaxime for 4–6 weeks (E) if gonococcal arthritis or a Gram-negative infection is suspected. The BNF is a good source of information for looking up the latest guidelines regarding antibiotic treatment regimens for common types of infection. Cefotaxime is a third generation cephalosporin. Cephalosporins are part of the beta-lactam group of antibiotics which work by inhibiting cell wall synthesis. The penicillins are also part of this group. There are different generations of cephalosporins, with those of later generations having increasing Gram-negative but decreasing Gram-positive cover. Cefotaxime is also used to treat meningitis and gonorrhoea. Some of the other commonly used third generation cephalosporins are ceftizoxime and ceftriaxone – you can remember these because they all have a ‘t’ in their names, just like in ‘third’ generation.

How well did you know this?
1
Not at all
2
3
4
5
Perfectly
42
Q

You order hepatitis B serology tests for one of your patients, a 24-year-old man who is an intravenous drug user. The results that come back from the laboratory are as follows: • HBsAg = positive • Anti-HBs = negative • HBeAg = positive • Anti-HBe = negative • Anti-HBc IgM = negative • Anti-HBc IgG = positive What is the most likely diagnosis based on these results? A The patient has chronic hepatitis B infection which is currently highly infectious B The patient has chronic hepatitis B infection which is not currently infectious C The patient has acute hepatitis B infection which is not currently infectious D The patient is immune due to hepatitis B vaccination E The patient is immune due to natural infection

A

A The patient has chronic hepatitis B infection which is currently highly infectious The HBsAg positive indicate that the patient has hepatitis B, and the HBeAg indicates that it is highly infectious (A). The anti-HBc IgG is also a marker that it is a chronic infection. The different hepatitis B surface antigens and antibodies can become quite confusing, but are often asked about in exam questions. Here is a summary of what you should know: • HBsAg – The ‘s’ stands for surface, and refers to a protein on the surface of the virus. It is the first detectable antigen to appear after someone has been infected, and can be positive in acute or chronic disease. Patients who still carry this antigen after 6 months are termed hepatitis carriers. It is this antigen that is used to make the hepatitis B vaccine • Anti-HBs – This is an IgG antibody that appears after the host has cleared the infection, and indicates recovery. It is also found in a person who has been vaccinated against hepatitis B (D) • HBeAg – the ‘e’ antigen is often used as a marker of infectivity, as it is only found in the blood when the virus is actively replicating. If you find this hard to remember, think of the ‘e’ standing for ‘eek! I’m infectious!’ If the patient was not infectious (B), this would not be present • Anti-HBc IgM – this indicates that the patient has recently been infected with hepatitis B, and is a marker of acute infection (C) • Anti-HBc IgG – this is produced in response to the core antigen, and often persists for life. You can remember this as the ‘c’ standing for ‘chronicity’, as it is the difference between IgM and IgG antibodies which can tell you whether the infection is acute or chronic. And to remember which way round it is, think of ‘My Gosh, he’s chronic!’ If the patient was immune from natural infection (E), HBsAg would not be positive, but anti-HBc IgG would be.

How well did you know this?
1
Not at all
2
3
4
5
Perfectly
43
Q

You order hepatitis B serology tests for one of your patients, a 24-year-old man who is an intravenous drug user. The results that come back from the laboratory are as follows: • HBsAg = negative • Anti-HBs = positive • HBeAg = negative • Anti-HBe = negative • Anti-HBc IgM = negative • Anti- HBc IgG = negative What is the most likely diagnosis based on these results? A The patient has chronic hepatitis B infection which is currently highly infectious B The anti-HBs is a false positive result C The patient has a resolved hepatitis B infection D The patient is immune due to hepatitis B vaccination E The patient is immune due to natural infection

A

D The patient is immune due to hepatitis B vaccination Remember from the previous question that the anti-HBs antibody appears after the host has cleared the infection, and indicates recovery. It is also found in a person who has been vaccinated against hepatitis B (D). If you get an exam question which only has the anti-HBs positive, think of vaccination! Levels of this antibody are measured to see if the patient has responded adequately to the vaccine.

How well did you know this?
1
Not at all
2
3
4
5
Perfectly
44
Q

A 79-year old woman is admitted to the hospital for treatment of pneumonia and is commenced on intravenous antibiotic therapy. Her respiratory symptoms begin to improve, but 5 days later she develops profuse diarrhoea. The most appropriate treatment is: A Oral metronidazole for 7 days B Oral metronidazole for 14 days C Isolation and treatment with intravenous fluids D IV metronidazole for 7 days E Oral co-amoxiclav for 7 days

A

B Oral metronidazole for 14 days Broad spectrum antibiotics, such as those used for pneumonia, can eradicate a patient’s normal gut flora and therefore increase their susceptibility to Clostridium difficile infection. This is particularly true of penicillin derivatives (as was most likely used to treat her pneumonia), clindamycin, and third generation cephalosporins. It classically presents with profuse watery diarrhoea, usually of acute onset. The most common time for it to occur is 4–9 days after the antibiotics are started, but it can occur up to 2 months after discontinuing treatment. Clostridium difficile is a Gram-positive, anaerobic rod-shaped bacterium. The gold standard for diagnosis is detection of the C. difficile toxin in a stool sample.

How well did you know this?
1
Not at all
2
3
4
5
Perfectly
45
Q

A 79-year old woman is admitted to hospital for treatment of pneumonia and is commenced on intravenous antibiotic therapy. Her respiratory symptoms begin to improve, but 5 days later she develops profuse diarrhoea. After treatment with oral metronidazole she shows gradual improvement, but the profuse diarrhoea returns 2 weeks later. The same organism is found to be responsible. The most appropriate course of action is: A Oral metronidazole for 7 days B Oral metronidazole for 14 days C Isolation and treatment with intravenous fluids D IV metronidazole for 7 days E Oral vancomycin for 14 days

A

B Oral metronidazole for 14 days This patient’s repeated diarrhoea may be caused by persistent infection with Clostridium difficile (spore germination), new infection or resistant bacteria. Current guidelines recommend the use of a repeat course of metronidazole for the treatment of recurrent C. difficile infection (B). As explained previously, a 7-day course of metronidazole (A) is not considered a sufficient duration of treatment to eradicate the bacterium. Again, isolation and IV fluid resuscitation (C) is necessary but not adequate as a single measure in the management of this woman. Intravenous metronidazole (D) is only needed if a patient is not responding to vancomycin, the infection is life-threatening, or for patients with ileus. Oral vancomycin for 10–14 days (E) is given for: • Third or subsequent episodes • Severe infection • Infection not responding to metronidazole • patients who cannot tolerate metronidazole

46
Q

A 65-year old retired mechanic is brought by his family to his GP due to their concern over his recent increase in confusion. This has occurred rapidly over the past 4 months, and he now struggles to recognize members of his family. His daughter also reports occasionally seeing intermittent, jerky movements of both his arms. The GP organizes a CT scan and dementia screen, which are both found to be normal. Which is the next most useful diagnostic test for the GP to order? A MRI brain B Electroencephalogram C Electrocardiogram D Ultrasound scan of both carotids E Tonsillar biopsy

A

B Electroencephalogram The key here is the rapidly progressive nature of the condition in a relatively young patient. He shows the characteristic sudden decline in cognitive function, combined with the presence of myoclonic jerks and the lack of positive investigation results so far. This is highly suggestive of sporadic Creutzfeldt–Jakob disease (CJD), the name given to a common group of prion diseases. The word prion is derived from the words ‘protein’ and ‘infection’, and it so follows that a prion is a highly infectious agent composed of protein. There are essentially three different forms of CJD which you should be aware of: 1 Sporadic Creutzfeldt–Jakob Disease (80 per cent) 2 Acquired (

47
Q

A 61-year-old patient has recently been diagnosed with sporadic CJD. His GP is keen to do a lumbar puncture. Which of the following statements is true regarding this investigation in this situation? A The lumbar puncture is used to look for the levels of protein, glucose and polymorphs B The lumbar puncture is used to look for the levels of a protein called 14-3-3 C A lumbar puncture is the most specific test for variant CJD D The lumbar puncture is not useful in sporadic CJD, but is an important test in variant CJD E A tonsillar biopsy would be a more useful test than a lumbar puncture for sporadic CJD

A

B The lumbar puncture is used to look for the levels of a protein called 14-3-3 The lumbar puncture in CJD is used to analyze the CSF for a protein named ‘14-3-3’ (B). Note that routine analysis of the cerebrospinal fluid (CSF) is normal in CJD, therefore looking at levels of protein, glucose and polymorphs (A) would not be useful to distinguish between possible causative agents of the clinical features as it is in meningitis.

48
Q

A 16-year-old student complains of a headache of recent onset at school. He is taken to accident and emergency and on examination has a temperature of 37.6°C. A lumbar puncture is performed, and the results are as follows: • Appearance: Clear fluid • Protein: 0.82 g/L • WCC: 90.5 × 107 (>95 per cent lymphocytes) What is the most likely diagnosis? A Subarachnoid haemorrhage B Tension headache C Bacterial meningitis D Viral meningitis E Tuberculous meningitis

A

E Tuberculous meningitis In this context, the two most immediately worrying diagnoses for the onset of an acute headache are a subarachnoid haemorrhage and bacterial meningitis as both of these may be fatal if rapid intervention does not occur. Cause Appearance Neutrophil count (x 106/L) Lymphocyte count (x 106/L) Protein (g/L) Glucose (mmol/L) Normal Clear 0

49
Q

A 42-year-old alcoholic is admitted with abdominal distension. The shifting dullness test is positive and he is found to have diffuse abdominal tenderness. His observations are as follows: pulse 115, blood pressure 116/83, temperature 37.9°C. The next best course of action is: A Begin therapeutic paracentesis B Observe, administer analgesia and closely monitor his vital signs C Commence intravenous spironolactone D Commence intravenous amoxicillin E Commence intravenous cefotaxime

A

E Commence intravenous cefotaxime This patient is presenting with features suggestive of spontaneous bacterial peritonitis (SBP), which is a form of peritonitis in the absence of a contiguous source of infection. This usually results from the development of portal hypertension in patients with chronic liver disease. This group of patients are particularly susceptible as they are often immunocompromised. The pyrexia and tachycardia, in conjunction with the clinical features of abdominal tenderness and ascites, make this the most likely diagnosis in this patient. Other typical clinical features might include nausea, vomiting, confusion, general malaise or features of hepatic encephalopathy. In approximately 15 per cent of patients SPB can be asymptomatic. The most common organisms isolated in patients with SBP include E. coli, Gram-positive cocci and enterococci. Although local antibiotic guidelines may differ, of the options listed cefotaxime (E) is one of the most extensively studied and has been proven to be effective.

50
Q

A 63-year-old asymptomatic housewife is referred to a gastroenterologist after her GP found that she had abnormal liver function tests on a routine blood test. A thorough history reveals that she received a blood transfusion during her pregnancy in 1979. Further tests confirm that she has contracted hepatitis C. She is commenced on a course of anti-viral treatment. Which of the following factors is most significant in influencing her chance of clearing the virus? A The length of time between contracting the disease and being diagnosed B The route by which she contracted the disease C Her liver function test results D The virus genotype E The level of alpha-feto-protein

A

D The virus genotype Hepatitis C is a single stranded RNA virus that is similar in structure to the ‘flaviviruses’. It can cause a slowly progressive disease of the liver that is frequently asymptomatic and which cannot be vaccinated against. Routes of transmission include: • blood products (before 1991, when screening of blood donors for the disease was introduced) • intravenous drug use • sexual transmission • vertical transmission • less commonly: needle-stick injuries, tattoos

51
Q

A 63-year-old asymptomatic housewife is referred to a gastroenterologist after her GP found that she had abnormal liver function tests on a routine blood test. A thorough history reveals that she received a blood transfusion during her pregnancy in 1979. The best test to confirm whether the patient has hepatitis C would be: A Liver biopsy B Anti-hepatitis C antibodies C Alanine aminotransferase levels D Hepatitis C RNA PCR E Viral genotyping

A

D Hepatitis C RNA PCR Hepatitis C RNA PCR (D) – This can be used to differentiate between a current and past infection. A quantitative test to detect the number of hepatitis C RNA particles (called the ‘viral load’) can also be performed. This can be very useful to detect a patient’s response to the anti-viral treatment. Therefore, this is the best diagnostic test for hepatitis C

52
Q

A 33-year-old backpacker visits his GP complaining of feeling weak, lethargic and feverish since he returned from his trip to South Africa 3 months previously. He is accompanied by his wife, who reports a change in his behaviour and disturbed sleeping pattern since his return. On examination, his GP discovers that he has enlarged cervical lymph nodes, and there is a small chancre on his forearm that is approximately 2 cm in diameter. The most likely causative organism is: A Plasmodium falciparum B Trypanosoma brucei gambiense C Trypanosoma brucei rhodesiense D Trypanosoma cruzi E Leishmania infantum

A

C Trypanosoma brucei rhodesiense Human African trypanosomiasis is also known as sleeping sickness, and is an infection transmitted by the tsetse fly in sub-Saharan Africa. There are two main types: Trypanosoma brucei rhodesiense (C) is found in south and eastern Africa, accounts for under 5 per cent of cases, and causes an acute infection with symptoms appearing over a few weeks or months. You can remember this as rhodesiense causes a rapid infection. As this patient’s symptoms appeared 3 months after returning from his travels, this is more likely to be the causative agent here 1 Trypanosoma brucei gambiense (B) is found in west and central Africa, is responsible for over 95 per cent of cases, and causes a chronic infection. It can take months or even years for symptoms to appear. You can remember this as gambiense causes a gradual infection

53
Q

A 20-year-old student seeks medical attention due to recent difficulty in swallowing, and severe weight loss. A thorough travel history reveals that he returned several months ago from a gap year in Brazil. During his trip he remembers becoming unwell at one point with a fever, diarrhoea, vomiting and swollen eyelids, but this resolved in approximately 3 weeks with no treatment. A chest x-ray is ordered as one of his investigations, and this reveals marked dilatation of his oesophagus. The vector responsible for transmitting this disease is: A Tsetse fly B Reduviid bug C Sandfly D Aedes mosquito E Ixodes tick

A

B Reduviid bug Trypanosoma cruzi is responsible for causing Chagas disease, a potentially life-threatening disease which is spread by reduviid bugs (B) in Brazil. These are also known as ‘kissing bugs’. A red nodule, called a chagoma, can appear at the site of the bite. There are two forms of the disease: acute and chronic. In the acute phase, patients may experience non-specific symptoms such as fever, lethargy, diarrhoea, and vomiting. A characteristic feature, but one which occurs in less than 50 per cent of cases, is a purplish swelling of the eyelids (called Romana’s sign). To put this all together, picture Tom Cruise (Trypanosoma cruzi) starring in a gladiator film as a Roman (Romana’s sign) wearing purple sunglasses (swollen eyelids) and being kissed (kissing bugs) by lots of fans ‘ready with their video cameras’ (reduviid!)

54
Q

A 46-year-old Somalian woman presents to her GP with a dry cough and weight loss of 5 kg over 3 weeks. She is sent to the hospital, and a chest x-ray reveals cavitating lung lesions. The most appropriate therapy is: A Rifampicin and isoniazid for 6 months, ethambutol and pyrazinamide for 2 months B Rifampicin and isoniazid for 2 months, ethambutol and pyrazinamide for 6 months C Rifampicin and pyrazinamide for 4 months, ethambutol and isoniazid and for 2 months D Rifampicin and streptomycin for 4 months, pyrazinamide and ethambutol for 2 months E Rifampicin, isoniazid, ethambutol and pyrazinamide for 6 months

A

A Rifampicin and isoniazid for 6 months, ethambutol and pyrazinamide for 2 months Current guidelines in the UK recommend the following antibiotic treatment for pulmonary tuberculosis: • Isoniazid and rifampicin for 6 months • Pyrazinamide and ethambutol for the first 2 months

55
Q

A 46-year-old Somalian woman presents to her GP with a dry cough and weight loss of 5 kg over 3 weeks. She is sent to the hospital, and a chest x-ray reveals cavitating lung lesions. She is started on a course of anti-tuberculous medication. Which of the following statements about this regimen is true? A Liver function tests only need to be checked in those with pre-existing liver disease B Ethambutol can cause a peripheral neuropathy C Pyridoxine should always be given with isoniazid treatment D Rifampicin can cause optic neuritis E Ethambutol should be avoided in renal failure

A

E Ethambutol should be avoided in renal failure Remember that treatment for pulmonary TB usually consists of two phases – an initial phase with rifampicin, isoniazid, pyrazinamide and ethambutol for 2 months, and then a continuation phase with rifampicin and isoniazid only for 4 months. Streptomycin and ethambutol are two anti-tuberculous drugs which should preferably be avoided in patients with renal impairment ( E). If they have to be used the dosage should be reduced and the plasma drug concentration closely monitored. A patient’s renal function should be checked routinely before anti-tuberculous medication is started. The side effect that is particularly worrying with the use of ethambutol is its ocular toxicity, and this is more likely in renal impairment as it is renally excreted. This can present with changes in visual acuity, colour blindness and restriction of visual fields. Therefore a patient’s visual acuity should be assessed with a Snellen chart prior to starting treatment, and they should be strongly advised to stop the medication and seek advice if they become aware of any change in their vision

56
Q

A 35-year-old banker develops a fever, vomiting and diarrhoea after a barbeque. This resolves within 2 weeks, but he then suddenly develops unilateral facial weakness. This is followed by severe muscle weakness which rapidly spreads over the next 5 days from his feet and legs to his trunk. The most likely diagnosis is: A Polio B Lyme disease C Guillan–Barré syndrome D Haemolytic uraemic syndrome E Influenza

A

C Guillan–Barré syndrome This scenario is characteristic of Guillan–Barrè syndrome. If you remember that this disease is also known as AIDP – acute inflammatory demyelinating polyradiculopathy – you can remember the underlying pathology more easily. It is usually triggered by an infection, and it is thought that a suppressed T-cell response results in an immunological reaction that targets the peripheral nerves. The triggering infection is most commonly Campylobacter jejuni (as alluded to here), but other common causes can include Mycoplasma pneumoniae and viruses such as cytomegalovirus and influenza

57
Q

A young girl returns from visiting her relatives in India, feeling feverish and with flu-like symptoms. A diagnosis of malaria is suspected. Her fevers started on Monday, regressed for a few days and then returned on Thursday. She was well again over the weekend, and was then brought to the GP the following Monday when her fever had again returned. The most likely causative agent in this case is: A Plasmodium falciparum B Plasmodium vivax C Plasmodium ovale D Plasmodium malariae E Plasmodium knowlesi

A

D Plasmodium malariae Plasmodium falciparum (A) 7–14 days Malignant tertian This is the most severe form Plasmodium vivax (B) 12–17 days Benign tertian Relapse can occur with these forms because the parasite can lie dormant in the liver, and can produce symptoms months or years later Plasmodium ovale (C) 15–18 days Benign tertian Plasmodium malariae (D) 18–40 days Benign quartan Relapse can occur with this too, but this time the parasites lie dormant in the blood Plasmodium knowlesi (E) 12 days Quotidian (daily) This form mainly occurs in southeast Asia (such as in Borneo), and not in Africa. It does not normally relapse

58
Q

A young girl returns from visiting her relatives in India, feeling feverish and with flu-like symptoms. A diagnosis of malaria is suspected. The form of the malaria parasite which invades erythrocytes is known as a: A Sporozite B Schizont C Merozite D Hypnozoite E Gametocyte

A

C Merozite The merozites escape from the liver into the blood stream and infect red blood cells – the erythrocytic phase

59
Q

A 55-year-old housewife returns from visiting her relatives in India, with a high fever and with flu-like symptoms. A diagnosis of uncomplicated falciparum malaria is confirmed. The most appropriate management plan is: A Discharge with oral quinine and doxycycline B Discharge with oral mefloquine and chloroquine C Admit, give IV paracetemol and observe D Admit and give IV quinine E Admit and give oral quinine and doxycycline

A

E Admit and give oral quinine and doxycycline Uncomplicated malaria can be treated with one of the following: 1 Oral quinine plus doxycycline for 5–7 days (E) 2 Co-artem (artemetherelumefantrine) for 3 days 3 Atovaquone–proguanil (Malarone) for 3 days

60
Q

A 55-year-old housewife returns from visiting her relatives in India, with a high fever and with flu-like symptoms. thick and thin films are requested, and Maurer’s clefts are seen under the microscope. The diagnosis is: A Plasmodium falciparum B Plasmodium vivax C Plasmodium ovale D Plasmodium malariae E Plasmodium knowlesi

A

A Plasmodium falciparum The most reliable way to diagnose malaria is via a blood film, and traditionally a thick and thin blood film are requested. Most people remember this fact, but not the reason behind it! Thick films are better than thin films at picking up lower levels of infection, but thin films allow the specific species to be identified. Both types of films are used together to make the diagnosis. In the erythrocytic life cycle of the malarial parasite, disc-like granulations can be seen at the edge of the cell using an electron microscope. These are known as Maurer’s clefts, and are found in falciparum malaria (A).

61
Q

A Streptococcus pneumoniae B Moraxella catarrhalis C Haemophilus influenzae D Legionella pneumophila E Mycoplasma pneumonia F Chlamydia pneumoniae G Mycobacterium tuberculosis H Pneumocystis jirovecii I Staphylococcus aureus

A 25-year-old man with a history of recurrent chest infections presents to an infectious disease specialist. A subsequent chest X-ray demonstrates widespread pulmonary infiltrates. A sputum stain using Gomori’s methenamine silver reveals characteristic cysts.

A

H Pneumocystis jirovecii Pneumocystis jirovecii (H) is a yeast-like fungus that primarily affects immunocompromised patients such as those with HIV. Pneumocystis pneumonia may be the presenting feature of HIV and patients with a CD4 count less than 200 cells/μL are particularly susceptible. Clinically, Pneumocystis jirovecii infection presents with fever, non-productive cough, weight loss and night sweats. Chest X-ray may show signs of diffuse bilateral pulmonary infiltrates. Definitive diagnosis involves histological examination of sputum or bronchio-alveolar lavage fluid. Gomori’s methenamine silver stain reveals ‘flying saucer’ shaped cysts on microscopy.

62
Q

A Streptococcus pneumoniae B Moraxella catarrhalis C Haemophilus influenzae D Legionella pneumophila E Mycoplasma pneumonia F Chlamydia pneumoniae G Mycobacterium tuberculosis H Pneumocystis jirovecii I Staphylococcus aureus

A 54-year-old woman admitted to the respiratory ward is found to have right sided consolidation on chest X-ray. Histological examination reveals Grampositive cocci arranged in pairs.

A

A Streptococcus pneumoniae Streptococcus pneumoniae (pneumococci; A) are α-haemolytic Grampositive cocci arranged in pairs (diploccoci). As Streptococcus pneumoniae are capsulated bacteria, the Quelling reaction in which pneumococci are mixed with anti-serum and methylene blue causes the capsule to swell can be visualized under the microscope. Optochin-sensitivity also differentiates pneumococcus from Streptococcus viridans (also α-haemolytic), which is optochin-insensitve. Clinically, lobar consolidation is visible on X-ray, which represents a collection of pus, bacteria and exudate in the alveoli.

63
Q

A Streptococcus pneumoniae B Moraxella catarrhalis C Haemophilus influenzae D Legionella pneumophila E Mycoplasma pneumonia F Chlamydia pneumoniae G Mycobacterium tuberculosis H Pneumocystis jirovecii I Staphylococcus aureus

A 65-year-old woman is brought into accident and emergency with severe respiratory distress. The patient’s history revealed that she had been seen by her GP due to a viral infection 2 weeks previously. Histological examination reveals Gram-positive cocci arranged in clusters.

A

I Staphylococcus aureus Staphylococcus aureus (I) are β-haemolytic Gram-positive cocci arranged in grape-like clusters. All staphylococci are also catalase positive, whereas streptococci are catalase negative. Clinically, S. aureus can cause consolidation, cavitations of the lungs and empyema (pus in the pleural space). Staphylococcus aureus has a number of virulence factors including anti-immune proteins (haemolysins, leukocidins and penicillinase) as well as tissue break-down proteins (hyaluronidase, staphylokinase and protease).

64
Q

A Streptococcus pneumoniae B Moraxella catarrhalis C Haemophilus influenzae D Legionella pneumophila E Mycoplasma pneumonia F Chlamydia pneumoniae G Mycobacterium tuberculosis H Pneumocystis jirovecii I Staphylococcus aureus A 40-year-old HIV positive man is seen by his GP. The patient admits a 4-week history of cough. The GP requests acid-fast staining of the patient’s sputum.

A

G Mycobacterium tuberculosis Mycobacterium tuberculosis (G) is an acid-fast bacillus which is transmitted via aerosol droplets. Clinical manifestations include fever, cough (with possible haemoptysis), weight loss and night sweats. Tuberculosis is highly prevalent in HIV patients due to impaired cell-mediated immunity. Chest X-ray reveals bihilar lymphadenopathy. Most commonly, Ziehl–Neelson staining is performed on a sputum sample demonstrating acid-fast bacilli, but auramine–rhodamine staining can also be used. Mycobacterium tuberculosis, however, take approximately 6 weeks to culture, and hence faster polymerase chain reaction diagnostic tests are being developed.

65
Q

A Streptococcus pneumoniae B Moraxella catarrhalis C Haemophilus influenzae D Legionella pneumophila E Mycoplasma pneumonia F Chlamydia pneumoniae G Mycobacterium tuberculosis H Pneumocystis jirovecii I Staphylococcus aureus A 36-year-old engineer presents to his GP with a 1-week history of headache, myalgia and cough. Blood tests reveal hyponatraemia. A urinary antigen test is found to be positive.

A

D Legionella pneumophila Legionella pneumophila (D) is an aerobic Gram-negative rod which causes an atypical pneumonia. It primarily affects those who work with air-conditioning units and can lead to milder Pontiac fever or more severe Legionnaire’s disease. Clinical features of legionellosis are nonspecific and may include headache, myalgia, confusion, rhabdomyolysis and abdominal pain. Blood chemistry may reveal hyponatraemia, hypophosphataemia and/or deranged liver enzymes. Diagnosis involves culture of respiratory secretions on buffered charcoal yeast extract agar, although a rapid urinary antigen test can also be used.

66
Q

A Vibrio cholerae B Staphylococcus aureus C Enterobacteriaecae D Listeria monocytogenes E Salmonella enteritidis F Shigellae G Campylobacter jejuni H Giardia lamblia I Entamoeba histolytica A 34-year-old HIV-positive woman is seen in the GP clinic due to 3 days of diarrhoea, headaches and fever. History reveals the patient had recently drunk unpasteurized milk. The causative organism is found to be β-haemolytic with tumbling motility.

A

D Listeria monocytogenes Listeria monocytogenes (D) is a β-haemolytic anaerobic Gram-positive rod that may cause outbreaks of non-invasive gastroenteritis. Sources include refrigerated food and unpasteurized dairy products. Clinical features of listeria infection include watery diarrhoea, abdominal cramps, headaches and fever, but minimal vomiting. Listeria demonstrates ‘tumbling motility’ as a result of flagellar-driven movements. Neonates and immunocompromised patients are particularly susceptible. Invasive infection can cause more serious problems in these groups including septicaemia, meningitis and encephalitis.

67
Q

A Vibrio cholerae B Staphylococcus aureus C Enterobacteriaecae D Listeria monocytogenes E Salmonella enteritidis F Shigellae G Campylobacter jejuni H Giardia lamblia I Entamoeba histolytica A 10-year-old girl has just returned from a summer swimming camp at Lake Windermere. She presents to accident and emergency with bloody diarrhoea and abdominal pain. Blood tests reveal anaemia and thrombocytopenia.

A

C Enterobacteriaecae Escherichia coli (C) is a Gram-negative rod-shaped bacterium that is a common cause of traveller’s diarrhoea in those returning from abroad. Transmission occurs via food and water that become contaminated with human faeces, as can swimming in contaminated lakes. Enterohaemorrhagic E. coli infection (serotype O157:H7) can lead to haemolytic uraemic syndrome (HUS), characterized by haemolytic anaemia, acute renal failure (uraemia) and a low platelet count (thrombocytopenia). Other diarrhoea-causing strains of E. coli include enterotoxigenic, enteropathogenic and enteroinvasive forms.

68
Q

A Vibrio cholerae B Staphylococcus aureus C Enterobacteriaecae D Listeria monocytogenes E Salmonella enteritidis F Shigellae G Campylobacter jejuni H Giardia lamblia I Entamoeba histolytica An 18-year-old on his gap year in India suddenly develops severe watery diarrhoea. Microscopy of his stool reveals no leukocytes but rods with fast movements.

A

A Vibrio cholerae Vibrio cholerae (A) are comma-shaped oxidase positive bacteria, causing profuse watery diarrhoea containing no inflammatory cells on microscopy. Transmission occurs via the faecal-oral route. Vibrio cholerae colonizes the small intestinal section of the gut and secretes enterotoxin containing subunits A (active) and B (binding). B subunit binds to GM1 ganglioside on the intestinal epithelial cells. Intracellularly, there is activation of cAMP by A subunit, which causes active secretion of sodium and chloride ions; as a consequence water is lost due to the osmotic pull of NaCl.

69
Q

A Vibrio cholerae B Staphylococcus aureus C Enterobacteriaecae D Listeria monocytogenes E Salmonella enteritidis F Shigellae G Campylobacter jejuni H Giardia lamblia I Entamoeba histolytica A 25-year-old homosexual man presents to his GP with a 3-day history of foul smelling, non-bloody diarrhoea, with abdominal cramps and flatulence. Stool microscopy reveals pear-shaped organisms.

A

H Giardia lamblia Giardia lamblia (H) is a pear-shaped trophozite containing two nuclei, four flagellae and a suction disc. Transmission occurs via ingestion of a cyst from faecally contaminated water and food. Trophozites attach to the duodenum but do not invade. Instead, protein absorption is inhibited, drawing water into the lumen of the gastrointestinal tract. G. lamblia must be considered in travellers, hikers and homosexual men. Clinically, foul smelling non-bloody steatorrhoea is produced, with stool containing cysts visible on microscopy.

70
Q

A Vibrio cholerae B Staphylococcus aureus C Enterobacteriaecae D Listeria monocytogenes E Salmonella enteritidis F Shigellae G Campylobacter jejuni H Giardia lamblia I Entamoeba histolytica A 35-year-old woman presents to accident and emergency with fever, diarrhoea and signs of shock. Her husband mentions that she had attended a work colleague’s barbeque the previous day. The consultant believes superantigens are responsible for the patient’s condition.

A

B Staphylococcus aureus Staphylococcus aureus (B) are β-haemolytic Gram-positive cocci arranged in grape-like clusters. In the gastrointestinal tract, S. aureus produces the exotoxin TSST-1, which acts as a superantigen causing non-specific activation of T cells and subsequent release of IL-1, IL-2 and TNF-α. A massive non-specific immune response follows causing shock and multiple organ failure. Enterotoxin produced by bacteria causes vomiting and diarrhoea 12–24 hours after the culprit food has been consumed.

71
Q

A Neisseria meningitides B Herpes simplex virus-2 C Leptospira interrogans D Listeria monocytogenes E Cryptococcus neoformans F Escherichia coli G Streptococcus pneumoniae H Borrelia burgdorferi I Mycobacterium tuberculosis A 45-year-old man presents to his GP with a 2-month history of headache. After a CT scan demonstrates an opacity, a lumbar puncture is performed and cerebrospinal fluid (CSF) analysis reveals a protein level of 4.5 g/L (0.15–0.4), lymphocyte count 345 (1–5) and glucose 4.0 mmol/L (2.2–3.3).

A

I Mycobacterium tuberculosis Mycobacterium tuberculosis (I) may lead to a subacute or chronic meningitis. Symptoms are non-specific, including fever, headache and confusion. Focal signs may be present as a result of a cerebral granuloma. A tuberculous granuloma that occurs in the cortex of the brain, subsequently rupturing into the subarachnoid space, is termed a Rich focus. Diagnosis of tuberculous meningitis involves a lumbar puncture; the CSF appears colourless and characteristically has high protein, low glucose and raised lymphocyte levels. Nucleic acid amplification tests as well as imaging studies (CT and MRI) can be useful in the diagnostic work-up.

72
Q

A Neisseria meningitides B Herpes simplex virus-2 C Leptospira interrogans D Listeria monocytogenes E Cryptococcus neoformans F Escherichia coli G Streptococcus pneumoniae H Borrelia burgdorferi I Mycobacterium tuberculosis A 26-year-old man has recently returned to the UK from a year of working in Africa where he was taking part in a charity farming project. He presents to accident and emergency with signs of meningism. A serological microscopic agglutination test is positive.

A

C Leptospira interrogans Leptospira interrogans (C) causes leptospirosis (also known as Weil’s syndrome). Transmission occurs via contact with animals. Leptospira are thin aerobic spirochaetes that are tightly coiled. The first stage of infection is known as the leptospiramic phase, during which the patient suffers non-specific symptoms such as fever, headache, malaise and photophobia. In the second immune phase, IgM antibodies have formed and meningitis, liver damage (causing jaundice) and renal failure may develop. CSF examination will reveal a raised white cell count. The microscopic agglutination test is considered the gold standard for diagnosing leptospirosis.

73
Q

A Neisseria meningitides B Herpes simplex virus-2 C Leptospira interrogans D Listeria monocytogenes E Cryptococcus neoformans F Escherichia coli G Streptococcus pneumoniae H Borrelia burgdorferi I Mycobacterium tuberculosis A 19-year-old woman who has recently started university is brought to accident and emergency with a headache and a spreading non-blanching rash. Gramstain of a blood sample reveals the presence of Gram-negative diplococci.

A

A Neisseria meningitides Neisseria meningitides (meningococcus; A) is a Gram-negative diplococcus. Infants aged 6 months to 2 years are most at risk as well as large numbers of adults living in close quarters. Virulence factors include its capsule (antiphagocytic), endotoxin (lipopolysaccharide causes haemorrhage from blood vessels resulting in characteristic petechiae in meningococcaemia) and IgA1 protease (destroys IgA). Neisseria meningitides can lead to meningitis (headache, photophobia and neck stiffness) and meningococcaemia (signs of sepsis with spreading petechial rash). Neisseria meningitides is grown best on Thayer–Martin VCN media (only allows Neisseria species to grow).

74
Q

A Neisseria meningitides B Herpes simplex virus-2 C Leptospira interrogans D Listeria monocytogenes E Cryptococcus neoformans F Escherichia coli G Streptococcus pneumoniae H Borrelia burgdorferi I Mycobacterium tuberculosis A 46-year-old man with a history of HIV presents to accident and emergency with neck stiffness, fever and severe photophobia. Examination of the CSF with India ink reveals yeast cells surrounded by halos.

A

E Cryptococcus neoformans Cryptococcus neoformans (E) is a polysaccharide encapsulated yeast that causes a subacute or chronic meningoencephalitis. It is transmitted by inhalation (the source of which is pigeon droppings). Cryptococcus neoformans is usually asyptomatic, but can be pathogenic in immunocompromised patients such as those with HIV. As well as meningitis, C. neoformans can also cause pneumonia, skin ulcers and bone lesions. Diagnosis is made by examination of CSF; India ink staining reveals yeast cells with a surrounding halo. Cryptococcal antigen test is, however, a more sensitive test.

75
Q

A Neisseria meningitides B Herpes simplex virus-2 C Leptospira interrogans D Listeria monocytogenes E Cryptococcus neoformans F Escherichia coli G Streptococcus pneumoniae H Borrelia burgdorferi I Mycobacterium tuberculosis A 35-year-old woman presents to her infectious disease specialist due to recurrent episodes of meningitis. During her last presentation CSF analysis reveals a protein level of 0.8 g/L (0.15–0.4), lymphocyte count 290 (0–5) and glucose 2.2 mmol/L (2.2–3.3).

A

B Herpes simplex virus-2 Herpes simplex virus 2 (HSV-2; B) is the most common cause of viral meningitis of all the herpes family. HSV-2 is transmitted via sexual contact or via the mother during birth. The virus infects mucosal epithelial cells or lymphocytes; retrograde transport occurs from peripheral nerves to ganglion. Viral causes of meningitis can be diagnosed on examination of CSF; it appears colourless, with a raised lymphocyte level, moderately raised protein and normal glucose concentration. Recurrent aseptic meningitis (Mollaret’s meningitis) can be caused by both HSV-1 and HSV-2.

76
Q

A Treponema pallidum B Klebsiella granulomatis C Neiserria gonorrhoeae D Trichomonas vaginalis E Candidia albicans F Chlamydia trachomatis G Bacterial vaginosis H Haemophilus ducreyi I Herpes simplex virus 2 A 28-year-old woman sees her GP complaining of fever, lower abdominal pain and painful intercourse. Vaginal swabs are sent for a nucleic acid amplification test which reveal sexually transmitted bacteria that can also cause lymphogranuloma venereum.

A

F Chlamydia trachomatis Chlamydia trachomatis (F) is a small Gram-negative obligate intracellular bacterium, causing the sexually transmitted infection chlamydiosis. It has an affinity towards columnar epithelia that line mucous membranes. Serovars D–K cause genital chlamydiosis (as well as opthalmia neonatorum) resulting in dyspareunia, dysuria and vaginal/penile discharge. Serovars L1, L2 and L3 cause lymphogranuloma venereum, defined by a painless papule or ulcer on the genitals which heals spontaneously; the bacteria migrate along regional lymph nodes leading to lymphadenopathy.

77
Q

A Treponema pallidum B Klebsiella granulomatis C Neiserria gonorrhoeae D Trichomonas vaginalis E Candidia albicans F Chlamydia trachomatis G Bacterial vaginosis H Haemophilus ducreyi I Herpes simplex virus 2 A 68-year-old man presents to his GP with a gumma on his nose. On examination, the patient is found to have pupils that accommodate to light but do not react. The man admits to unprotected sexual intercourse during his youth.

A

A Treponema pallidum Treponema pallidum (A) causes syphilis. Syphilis has three clinical stages: primary, secondary and tertiary. Primary syphilis is defined by a firm painless chancre that appears approximately 1 month after sexual contact and resolves within a few weeks. Secondary syphilis is a bacteri aemic stage during which a widespread rash forms with lymphadenopathy. Tertiary syphilis occurs decades after the primary infection and involves multiple organs: gummatous lesions on skin and bone, aneurysm of the aortic arch, peripheral neuropathy, tabes dorsalis and Argyll–Robertson pupils.

78
Q

A Treponema pallidum B Klebsiella granulomatis C Neiserria gonorrhoeae D Trichomonas vaginalis E Candidia albicans F Chlamydia trachomatis G Bacterial vaginosis H Haemophilus ducreyi I Herpes simplex virus 2 A 35-year-old man presents to an infectious disease specialist with a painful penile ulcer and associated unilateral lymphadenopathy of the inguinal nodes. A swab of the ulcer is cultured on chocolate agar.

A

H Haemophilus ducreyi Haemophilus ducreyi (H) is a Gram-negative coccobacillus that causes a tropical ulcer disease (chancroid) and is contracted by sexual transmission. Chancroid is characterized by a painful genital ulcer that leads to unilateral painful swollen inguinal lymph nodes. Infected lymph nodes may rupture releasing pus. The differential diagnosis for genital ulcers includes syphilis (painless ulcer with bilateral painless lymphadenopathy), herpes simplex virus 1 and 2 (vesicles that eventually break down) and lymphogranuloma venereum (slowly developing painless inguinal lymph nodes). Haemophilus ducreyi can be cultured on chocolate agar.

79
Q

A Treponema pallidum B Klebsiella granulomatis C Neiserria gonorrhoeae D Trichomonas vaginalis E Candidia albicans F Chlamydia trachomatis G Bacterial vaginosis H Haemophilus ducreyi I Herpes simplex virus 2 A 28-year-old woman sees her GP complaining of fever, lower abdominal pain and painful intercourse. A vaginal swab is taken and subsequent Gram-staining reveals Gram-negative diplococci.

A

C Neiserria gonorrhoeae Neiserria gonorrhoeae (gonococcus; C) is an intracellular Gramnegative diplococcus that causes gonorrhoea. Virulence factors allow gonococci to evade phagocytosis and adhere to the non-ciliated epithelium of the fallopian tubes. In both men and women N. gonorrhoeae causes urethritis which presents with dysuria and purulent discharge (with associated dyspareunia in women). Long-term complications include pelvic inflammatory disease in women and epididymitis, prostititis as well as urethral stricture in men. Systemic invasion of bacteria causes pericarditis, endocarditis, meningitis and/or septic arthritis. Diagnosis involves Gram stain and culture on Thayer–Martin VCN medium, or PCR.

80
Q

A Treponema pallidum B Klebsiella granulomatis C Neiserria gonorrhoeae D Trichomonas vaginalis E Candidia albicans F Chlamydia trachomatis G Bacterial vaginosis H Haemophilus ducreyi I Herpes simplex virus 2 A 35-year-old woman presents to her GP with a 2-week history of a fishy odorous vaginal discharge, which occurs especially after sexual intercourse. Microscopy of the discharge reveals clue cells.

A

G Bacterial vaginosis Bacterial vaginosis (BV; G) is caused by an imbalance in the naturally occurring bacterial flora of the vagina and is a condition associated with sexual activity (not transmitted). A ‘fishy’ smelling white–cream vaginal discharge is characteristically produced. Diagnosis involves obtaining vaginal swabs. A litmus test will indicate loss of acidity with a pH greater than 4.5 (normal vaginal pH = 3.8–4.2). If a sample of the discharge is visualized under a microscope with sodium chloride, clue cells will be seen.

81
Q

A Amoxicillin B Doxycycline C Co-amoxiclav IV D Meropenam E Chloramphenicol F Cefotaxime G Vancomycin H Trimethoprim I Flucloxacillin A 54-year-old man presents to his GP with a 1-week history of fever, cough and fatigue. On examination his respiratory rate is 20 breaths per minute and he is normotensive. Subsequent chest X-ray reveals right lower lobe consolidation.

A

A Amoxicillin Amoxicillin (A) is a β-lactam antibiotic which inhibits enzymes responsible for cell wall synthesis, leading to osmotic lysis of the bacteria. As a result, β-lactams are ineffective against bacteria that lack cell walls such as Mycoplasma spp. and Chlamydia spp. In this case, amoxicillin is the best choice antibiotic to treat mild community acquired pneumonia. It is also useful in the treatment of urinary tract infection (UTI), Listeria meningitis, endocarditis prophylaxis and protection against Streptococcus pneumoniae in asplenic patients. Major side effects can be divided into allergic (anaphylaxis) and non-allergic (Steven–Johnson syndrome) consequences.

82
Q

A Amoxicillin B Doxycycline C Co-amoxiclav IV D Meropenam E Chloramphenicol F Cefotaxime G Vancomycin H Trimethoprim I Flucloxacillin A 38-year-old man presents to accident and emergency with an inflamed and swollen right leg. He mentions that he had cut the same leg 2 days previously playing football. A swab of the area isolates Staphylococcus aureus.

A

I Flucloxacillin Flucloxacillin (I) is a β-lactam antibiotic that is especially effective against Gram-positive bacteria that produce β-lactamase, for example S. aureus. Just like amoxicillin, flucloxacillin inhibits cell wall synthesis. Indications for its use include staphylococcal skin infections such as cellulitis (in this case), folliculitis and mastitis as well as pneumonia (adjunct), osteomyelitis, septic arthritis, endocarditis and prophylaxis in surgery. A rare side effect of flucloxacillin is cholestatic jaundice which may develop weeks after treatment is stopped.

83
Q

A Amoxicillin B Doxycycline C Co-amoxiclav IV D Meropenam E Chloramphenicol F Cefotaxime G Vancomycin H Trimethoprim I Flucloxacillin A 34-year-old woman presents to her GP with lower abdominal pain and dysuria. A dipstick of her urine reveals the presence of protein, white cells and nitrites.

A

H Trimethoprim Trimethoprim (H) is an inhibitor of folate metabolism; it impairs synthesis of DNA by interfering with folic acid metabolism. It is used in the treatment of uncomplicated UTIs. Trimethoprim should be used with caution in patients with megaloblastic anaemia due to its interaction with folate. Side effects of trimethoprim include thrombocytopenia, megaloblastic anaemia and hyperkalaemia (via antagonism of sodium channels in the distal convoluted tubule of nephrons). Trimethoprim combined with another folate inhibitor, sulphamethoxazole, forms co-trimoxazole, which is used in the treatment of Pneumocystis jirovecii infection.

84
Q

A Amoxicillin B Doxycycline C Co-amoxiclav IV D Meropenam E Chloramphenicol F Cefotaxime G Vancomycin H Trimethoprim I Flucloxacillin A 56-year-old man is being cared for on the surgical ward after excision of a segment of his bowel after being diagnosed with colorectal carcinoma. The following day the surgical wound site is found to be inflamed. The patient has a fever and his blood pressure is slowly declining. Blood cultures reveal Grampositive cocci arranged in clusters that are resistant to β-lactam antibiotics.

A

G Vancomycin Vancomycin (G) is the drug of choice in cases of methicillin-resistant Staphylococcus aureus infections (MRSA). Vancomycin is a glycopeptide antibiotic that inhibits cell wall synthesis. It is too large to traverse the cell wall of Gram-negative bacteria and hence is primarily targeted to Gram-positive bacteria. Side effects include renal failure, ototoxicity, blood disorders, rash and anaphylaxis. Due to the potential side effects, serum drug levels must be monitored. Vancomycin is also a second-line antibiotic in the treatment of Clostridium difficile infection.

85
Q

A Amoxicillin B Doxycycline C Co-amoxiclav IV D Meropenam E Chloramphenicol F Cefotaxime G Vancomycin H Trimethoprim I Flucloxacillin An 18-year-old woman student presents to accident and emergency with headache, neck stiffness and photophobia. CT scan reveals no raised intracranial pressure. Gram-negative diploccoci are visualized on Gram-staining of the patient’s CSF.

A

F Cefotaxime Cefotaxime (F) is a third generation cephalosporin and is the drug of choice in treating Neisseria meningitidis, which is the most common cause of meningitis in the UK. Cefotaxime is a β-lactam antibiotic and therefore inhibits cell wall synthesis. If meningitis is suspected in the community, the patient should be started on benzyl-penicillin until they are transferred to a secondary care unit. Cefotaxime is also useful in the treatment of pyelonephritis, sepsis secondary to hospital acquired pneumonia and soft tissue infections.

86
Q

A Human immunodeficiency virus (HIV) B Epstein–Barr virus (EBV) C Hepatitis B virus D Cytomegalovirus (CMV) E Hepatitis D virus F Varicella zoster virus G Hepatitis C virus H Human herpes virus 8 I Influenza virus A 38-year-old man presents to his GP with vomiting, mild fever and loss of appetite. He admits to travelling to sub-Saharan Africa 2 months previously. On examination the patient is evidently jaundiced.

A

C Hepatitis B virus Hepatitis B virus (HBV; C) is a double-stranded DNA virus that is prevalent in sub-Saharan Africa. It is transmitted via sexual contact, contaminated blood products, intravenous drug use as well as vertical transfer from mother to child during child birth. The virus has an incubation period of 2–6 months with 80 per cent of infections remaining acute and 20 per cent becoming chronic with risk of cirrhosis and hepatocellular carcinoma. HBV antigens include HBsAg (surface antigen), HBcAg (core antigen) and HBeAg (soluble antigen).

87
Q

A Human immunodeficiency virus (HIV) B Epstein–Barr virus (EBV) C Hepatitis B virus D Cytomegalovirus (CMV) E Hepatitis D virus F Varicella zoster virus G Hepatitis C virus H Human herpes virus 8 I Influenza virus A 39-year-old homosexual man is referred to the gastroenterology department for an oesophogastroduodenoscopy (OGD) due to recent onset odynophagia. The OGD reveals multiple raised white plaques that can be removed by endoscopic scraping.

A

A Human immunodeficiency virus (HIV) Human immunodeficiency virus (HIV; A) possesses single-stranded RNA as well as enzymes (reverse transcriptase, integrase and protease) in its core. HIV is transmitted via sexual intercourse, blood products, intravenous drug use and vertically from mother to child. HIV infects CD4+ T cells; within the cell the RNA undergoes reverse transcription to make DNA which is integrated into the host DNA; the virus then becomes latent or buds to infect further. AIDS (CD4+ count

88
Q

A Human immunodeficiency virus (HIV) B Epstein–Barr virus (EBV) C Hepatitis B virus D Cytomegalovirus (CMV) E Hepatitis D virus F Varicella zoster virus G Hepatitis C virus H Human herpes virus 8 I Influenza virus A 15-year-old girl presents to her GP complaining of a sore throat, fever, fatigue and loss of appetite. A blood film demonstrates atypical lymphocytes and monospot test is positive.

A

B Epstein–Barr virus (EBV) Epstein–Barr virus (EBV; B) primarily infects B lymphocytes, binding via a complement receptor. Transmission involves person-to-person transfer through close contact. EBV is associated with glandular fever (infectious mononucleosis) which causes pharyngitis, lymphadenopathy, fever, splenomegaly and hepatomegaly. Rare sequelae include thrombocytopenia and erythema multiforme. EBV can also cause Hodgkin’s lymphoma (latent reactivation of EBV), Burkitt’s lymphoma and nasopharyngeal cancers. It is diagnosed on blood film (atypical lymphocytes), monospot test (positive heterophil antibody test) and/or EBV antibodies in the blood.

89
Q

A Human immunodeficiency virus (HIV) B Epstein–Barr virus (EBV) C Hepatitis B virus D Cytomegalovirus (CMV) E Hepatitis D virus F Varicella zoster virus G Hepatitis C virus H Human herpes virus 8 I Influenza virus A 68-year-old woman presents to her GP after a 3-day history of fever, cough, headache and nasal congestion. The doctor believes her symptoms are due to a virus that binds to sialic acid receptors.

A

I Influenza virus Influenza virus (I) is part of the orthomyxoviridae group of viruses and causes epidemics of influenza annually. The influenza virus causes primary pneumonia as well as delayed secondary bacterial pneumonia and otitis media in immunocompromised patients. It is a spherical virion with haemagglutinin (HA) and neuraminidase (NA) glycoproteins on the surface. HA binds to sialic acid receptors present in the upper respiratory tract; viral RNA is subsequently inserted into the host cell and HA is cleaved by clara cell tryptase. NA cleaves neuraminic acid, a component of protective mucin; as a result the protective barrier is disrupted exposing sialic acid receptor sites beneath. NA also has a role facilitating the release of newly formed influenza virions.

90
Q

A Human immunodeficiency virus (HIV) B Epstein–Barr virus (EBV) C Hepatitis B virus D Cytomegalovirus (CMV) E Hepatitis D virus F Varicella zoster virus G Hepatitis C virus H Human herpes virus 8 I Influenza virus A 55-year-old man who is being treated for lung cancer with chemotherapeutic agents sees his oncologist for a routine check-up. There is a rash in a dermatomal pattern on the patient’s forehead; the patient complains that there is a burning sensation in the distribution of the rash.

A

F Varicella zoster virus Varicella zoster virus (VZV; F) is a droplet-spread herpes virus that causes chickenpox in children and shingles in adults. Chickenpox is characterized by fever, malaise and a rash (erythematous base with fluid top) that spreads over the body. Complications include secondary bacterial infection and encephalitis. VZV remains dormant in the dorsal root ganglia and may reactivate in states of immunosuppression. The most common symptom is neuralgia which occurs in a dermatomal distribution; other manifestations include encephalitis, Guillain–Barré syndrome, facial palsy and progressive outer retinal necrosis.

91
Q

A Acyclovir B Oseltamivir C Interferon-α D Zidovudine E Gancylcovir F Lamivudine G Efivarenz H Ritonavir I Adamantadine A 40-year-old man presents to an infectious disease specialist with a 4-month history of weight loss, fever and malaise. On examination the patient has lymphadenopathy. His CD4 count is found to be 289 copies/μL. The patient is started on lamivudine, ritonavir and one other drug.

A

D Zidovudine Zidovudine (D) is a nucleoside reverse transcriptase inhibitor (NRTI) used in the treatment of HIV/AIDS (as well as prevention of vertical transmission from infected mothers). Treatment is commenced once the CD4 count falls below 350 copies/μL. Zidovudine works by inhibiting the action of the enzyme reverse transcriptase, preventing the conversion of HIV RNA to DNA, which consequently cannot be incorporated into the host DNA. Side effects include anaemia, neutropenia, hepatic and cardiac dysfunction as well as myopathy. The standard treatment regimen involves the use of two nucleoside reverse transcriptase inhibitors (NRTIs) and a non-nucleoside reverse transcriptase inhibitor (NNRTI; Efivarenz) or a protease inhibitor (PI; Ritonavir).

92
Q

A Acyclovir B Oseltamivir C Interferon-α D Zidovudine E Gancylcovir F Lamivudine G Efivarenz H Ritonavir I Adamantadine A 38-year-old intravenous drug user presents to an infectious disease specialist with a 1-week history of fever and malaise; on examination hepatomegaly is noted. The patient is found to be HBeAg positive and is subsequently commenced on lamivudine and one other drug.

A

C Interferon-α Interferon-α (IFN-α; C) is a protein that is used in the treatment of hepatitis B; it potentiates the immune system to fight active viral infection. IFN-α acts on the JAK-STAT pathway; IFN-α binds to the IFN-α receptor, causing phosphorylation of STAT1 and STAT2, which subsequently form a complex with IRF9 (a transcription factor), leading to the synthesis of anti-viral proteins. A NRTI and IFN-α is the standard treatment for hepatitis B infection. Pegylated-IFN-α is used in the treatment of hepatitis C; similar to IFN-α, the addition of polyethylene glycol increases the half life of the drug.

93
Q

A Acyclovir B Oseltamivir C Interferon-α D Zidovudine E Gancylcovir F Lamivudine G Efivarenz H Ritonavir I Adamantadine A 25-year-old man presents to his GP with a 3-day history of fever, cough, body aches and severe headaches. The patient is told to rest and drink plenty of fluids. However, he returns the following week stating his symptoms have not improved and is started on a drug that acts on viral neuraminidase.

A

B Oseltamivir Oseltamivir (B) is a viral neuraminidase inhibitor used in the treatment of influenza. Osteltamivir is in fact a pro-drug; once metabolized in the liver the active form GS4071 is produced. Once a newly formed influenza virion is produced, the surface viral protein haemagglutinin is bound to sialic acid receptors along the upper respiratory tract. Neuraminidase is normally responsible for cleaving the haemagglutinin–sialic acid receptor bond, hence facilitating the release of newly formed virions. Therefore, inhibiting neuraminidase activity prevents further viral replication.

94
Q

A Acyclovir B Oseltamivir C Interferon-α D Zidovudine E Gancylcovir F Lamivudine G Efivarenz H Ritonavir I Adamantadine A 3-year-old girl diagnosed with severe combined immunodeficiency is due to undergo a bone marrow transplant. She is given a drug as prophylaxis against cytomegalovirus infection.

A

E Gancylcovir Gancyclovir (E) is a 2´-deoxyguanosine analogue used in the treatment of cytomegalovirus (CMV) infection. It is the first line drug for the prophylaxis of CMV in bone marrow transplant patients. 2´-deoxyguanosine is phosphorylated to the triphosphate form, which prevents viral DNA polymerase from elongating viral DNA and therefore inhibits CMV replication. Gancyclovir can cause bone marrow toxicity; it may therefore be prescribed together with granulocyte-colony stimulating factor (G-CSF). Gancyclovir is also used in the treatment of human herpes virus 6 (HHV-6) and Epstein–Barr virus infection.

95
Q

A Acyclovir B Oseltamivir C Interferon-α D Zidovudine E Gancylcovir F Lamivudine G Efivarenz H Ritonavir I Adamantadine A 28-year-old woman presents to her GP with cold sores dotted across her lower lip. She is started on a medication that inhibits DNA polymerase function to speed the healing processes.

A

A Acyclovir Acyclovir (A) is a guanosine analogue anti-viral drug used primarily in the treatment of herpes simplex virus infections (HSV-1 and HSV-2). It is converted to acyclo-guanosine monophosphate (acyclo-GMP) by viral thymidine kinase. Acyclo-GMP is further phosphorylated to acycloguanosine triphosphate (acyclo-GTP). Acyclo-GTP is incorporated into the viral DNA strand, terminating the chain and stopping DNA polymerase from functioning. Aciclovir is also indicated for the treatment of varicella zoster, Epstein–Barr virus and cytomegalovirus infections (with decreasing efficacy).

96
Q

A Cryptoccus neoformans B Pityriasis versicolour C Aspergillus flavus D Histoplasma capsulatum E Phialophora verrucosa F Tinea capitis G Sporothrix schenckii H Tinea corporis I Candida albicans A 38-year-old man with known HIV presents to his GP with a 1-week history of white coloured creamy deposits inside his mouth. The patient is prescribed an oral nystatin wash.

A

I Candida albicans Candida albicans (I) can affect both immunocompetent and immunocompromised hosts. In the immunocompetent host, clinical features range from oral thrush (creamy-white patches with red base over mucous membranes of mouth; treated with nystatin) to vaginitis (vaginal inflammation, pruritis and discharge; speculum examination reveals patches of cottage cheese-like clumps fixed to vaginal wall). In immunocompromised patients, C. albicans infection leads to oesophagitis, characterized by odynophagia. Candidaemia can lead to severe flu-like symptoms and can be diagnosed by testing for blood β-D-glucan (a component of fungal cell walls).

97
Q

A Cryptoccus neoformans B Pityriasis versicolour C Aspergillus flavus D Histoplasma capsulatum E Phialophora verrucosa F Tinea capitis G Sporothrix schenckii H Tinea corporis I Candida albicans A 45-year-old man with known HIV presents to accident and emergency with headache, nausea, confusion and fever. Investigation of the patient’s CSF with India ink stain reveals yeast cells surrounded by a halo.

A

A Cryptoccus neoformans Cryptococcus neoformans (A) is an encapsulated yeast that is transmitted via inhaled spores from pigeon droppings. It is usually asymptomatic in most cases. Seventy-five per cent of cases occur in immunocompromised patients, characterized by the development of sub-acute or chronic meningitis. Cryptococcal meningitis is fatal without treatment due to the associated cerebral oedema and brainstem compression. Diagnosis is made by CSF analysis with India ink stain which reveals yeast cells surrounded by a halo (polysaccharide capsule). A cryptococcal antigen test can also be used which offers higher sensitivity.

98
Q

A Cryptoccus neoformans B Pityriasis versicolour C Aspergillus flavus D Histoplasma capsulatum E Phialophora verrucosa F Tinea capitis G Sporothrix schenckii H Tinea corporis I Candida albicans A 35-year-old woman presents to her GP with hyperpigmented spots on her back. Scrapings of the affected areas reveal a ‘spaghetti with meatballs’ appearance under the microscope.

A

B Pityriasis versicolour Pityriasis versicolor (B) is a chronic fungal infection caused by Malassezia furfur, characterized by hypopigmentation (in patients with dark skin tones) and hyperpigmentation (in patients with pale skin tones). Spots affect the back, underarm, arms, legs, chest, neck and rarely the face. Microscopic investigation of the M. furfur with potassium hydroxide reveals a ‘spaghetti with meatballs’ appearance. Wood’s light may also reveal an orange fluorescence in some cases.

99
Q

A Cryptoccus neoformans B Pityriasis versicolour C Aspergillus flavus D Histoplasma capsulatum E Phialophora verrucosa F Tinea capitis G Sporothrix schenckii H Tinea corporis I Candida albicans A 48-year-old HIV positive man who has recently migrated from sub-Saharan Africa presents to accident and emergency with chest pain, shortness of breath, fever and cough. A chest X-ray demonstrates a spherical opacity in the upper left lung field.

A

C Aspergillus flavus Aspergillus flavus (C) is a fungus that commonly grows on stored grains and can cause a spectrum of disease. Allergic reaction in the airways may cause allergic broncho-pulmonary aspergillosis (ABPA) which occurs due to an IgE mediated type I hypersensitivity reaction leading to bronchospasm and eosinophilia. Infection in pre-formed lung cavities (for example in TB patients) may lead to a fungal ball visible on chest X-ray (aspergilloma). Invasive aspergillosis is a chronic necrotizing infection that may occur in neutropenic patients (chemotherapy) or patients with end stage AIDS (CD4 count

100
Q

A Cryptoccus neoformans B Pityriasis versicolour C Aspergillus flavus D Histoplasma capsulatum E Phialophora verrucosa F Tinea capitis G Sporothrix schenckii H Tinea corporis I Candida albicans A 32-year-old gardener presents to his GP with small raised lesions on his left arm. He remembers working in a garden a few days previously which had been swamped with rose-thorns.

A

G Sporothrix schenckii Sporothrix schenckii (Rose garderner’s disease; G) is a fungus found in soil and plants that causes sporotrichosis. A prick by thorns causes nodular lesions to appear on the surface of the skin. Initially the lesions will be small and painless; left untreated they become ulcerated. Infection may also spread to joints, bone and muscle by this route. Inhalation of spores may lead to pulmonary disease and systemic infection may lead to central nervous system involvement. Treatment options include itraconazole, fluconazole and oral potassium iodide.

101
Q

A Psittacosis B Rabies C Brucellosis D Q fever E Leptospirosis F Mycobacterium marinium G Lyme disease H Cat scratch disease I Rocky mountain spotted fever A 45-year-old man has returned to the UK from a holiday to France. A week later he presents with flu-like symptoms, drenching sweats, a recurring fever and is beginning to complain of a lower back pain. He admits to have brought back some local cheeses on visits to regional farms.

A

C Brucellosis Brucellosis (C) is a Gram-negative rod-shaped bacterium that is harboured by cattle (Brucella abortus), goats (B. melitensis), pigs (B. suis) and dogs (B. canis). Brucella spp. are transmitted by inhalation, unpasteurized dairy produce and direct contact with animals. Symptoms include fever, myalgia, arthralgia, tiredness and in chronic cases may be associated with depression. Diagnosis is made by blood culture on Castaneda medium. Complications include granulomatous hepatitis (histology of liver biopsy demonstrates granulomata), endocarditis, oseteomyelitis and thrombocytopenia.

102
Q

A Psittacosis B Rabies C Brucellosis D Q fever E Leptospirosis F Mycobacterium marinium G Lyme disease H Cat scratch disease I Rocky mountain spotted fever A 36-year-old man presents to his GP with a painful right knee. He states that he visited the Prairie regions of Canada a month previous to this episode and states that his wife had mentioned there was a red rash on his back; on examination a target shaped rash is observed.

A

G Lyme disease Lyme disease (G) is caused by the spirochaete Borrelia burgdorferi which is transmitted by the Ixodes ticks harboured by certain species of mice and deer. Initial symptoms include erythema migrans (a spreading annular skin lesion with a characteristic target-shaped appearance), malaise, fever and musculoskeletal pain. Several weeks after the primary infection, the patient may experience neurological (headache, meningitis and Bell’s palsy) and cardiac (arrhythmias, myocarditis and pericarditis) effects. Late features include arthralgia and arthritis.

103
Q

A Psittacosis B Rabies C Brucellosis D Q fever E Leptospirosis F Mycobacterium marinium G Lyme disease H Cat scratch disease I Rocky mountain spotted fever A 38-year-old sewage worker presents to his GP with 1-week history of flulike symptoms with diarrhoea. A microscopic agglutination test reveals the diagnosis.

A

E Leptospirosis Leptospirosis (Weil’s disease; E) is a zoonotic disease caused by Leptospira interrogans which is harboured by both wild and domestic animals. It is transmitted via drinking water that has become contaminated with the urine of infected animals; as a result those involved in water-sports and sewage workers are at particular risk. Lyme disease is characterized by an influenza-like disease with/without gastrointestinal symptoms. Diagnosis can be made by ELISA, PCR or microscopic agglutination test (MAT). Long-term complications include hepatitis and renal failure.

104
Q

A Psittacosis B Rabies C Brucellosis D Q fever E Leptospirosis F Mycobacterium marinium G Lyme disease H Cat scratch disease I Rocky mountain spotted fever A 48-year-old man presents to his GP with flu-like symptoms. On examination the patient has a maculopapular rash on his trunk. The patient also shows an area where a vague bite mark is visible.

A

I Rocky mountain spotted fever Rocky Mountain spotted fever (I) is caused by Rickettsia spp. infection, a Gram-negative genus of bacteria, most prevalent in North and South America. It is harboured in small wild rodents and domestic animals (transmitted to humans by ticks). Rickettsia bacteria invade the endothelial lining of capillaries causing a vasculitis. Clinical features include headache, fever, myalgia, vomiting and confusion. Late signs include a rash that is maculopapular and/or petechial on the distal parts of the limbs which then spreads to the trunk and face. Rocky Mountain spotted fever may lead to thrombocytopenia, hyponatraemia and/or elevated liver enzymes.

105
Q

A Psittacosis B Rabies C Brucellosis D Q fever E Leptospirosis F Mycobacterium marinium G Lyme disease H Cat scratch disease I Rocky mountain spotted fever A 34-year-old bird handler presents to his GP with a few days’ history of fever, mild cough and myalgia. The patient states that his shop had recently taken a new shipment of parrots from Central America. Giemsa staining of the patient’s sputum reveals cytoplasmic inclusions.

A

A Psittacosis Psittacosis (A) is a zoonotic infectious disease caused by Chlamydia psittaci which is contracted from a wide variety of birds (parrots, pigeons and cockatiels to name a few). Human symptoms mainly involve a severe pneumonia (with or without hepatitis). Although the patient may report mild symptoms, the X-ray will generally appear to show severe pathology. Diagnosis is made by visualizing cytoplasmic inclusions on Giemsa or fluorescent antibody stained sputum or biopsy sample.

106
Q

A Rubella B Syphilis C Measles D Hepatitis B E Mumps F Listeria monocytogenes G Cytomegalovirus H Haemophilus influenzae I HIV A 10-year-old boy is brought to see the GP by his mother as he has recently developed parotid swelling associated with a fever. Blood tests reveal a raised amylase level. The boy’s mother reveals that his immunization schedule is not complete as they were living in Tunisia at the time.

A

E Mumps Mumps (E) is spread by droplets in the air which travel via the lungs to parotid tissue and subsequently to distant sites. Clinical features of infection consist of fever, malaise and transient hearing loss. Parotitis is characteristic of mumps infection with unilateral or bilateral swelling and pain on chewing. Plasma amylase levels may be elevated as a result of inflammation of the salivary glands. Complications such as viral meningitis, orchitis/oophoritis, mastitis and arthritis may result from long-standing infection. The MMR vaccine given at 12–18 months has drastically reduced the incidence of mumps.

107
Q

A Rubella B Syphilis C Measles D Hepatitis B E Mumps F Listeria monocytogenes G Cytomegalovirus H Haemophilus influenzae I HIV A 3-week-old baby develops vomiting and is feeding poorly. On examination he has a reduced level of consciousness and an arched back. Analysis of the CSF reveals the presence of Gram-positive rods.

A

F Listeria monocytogenes Listeria monocytogenes (F) is a β-haemolytic anaerobic Gram-positive rod that can cause meningitis in the neonate to 3 months age group. Listeria monocytogenes may be transmitted vertically from mother to baby in utero (due to the ingestion of infected food by the mother) or during birth (transvaginal transfer). Early signs of meningitis are nonspecific in the age group affected (fever, poor feeding, vomiting, seizures and reduced consciousness) whereas late signs include a bulging fontanelle, neck stiffness, opisthotonos (arched back), Brudzinski and Kernig signs positive as well as meningococcaemia.

108
Q

A Rubella B Syphilis C Measles D Hepatitis B E Mumps F Listeria monocytogenes G Cytomegalovirus H Haemophilus influenzae I HIV A 3-year-old girl presents to the GP with a cough, fever and runny nose. On examination, the child has white spots scattered on the buccal mucosa. Her mother admits that she denied her child a certain vaccine due to scares presented by the media.

A

C Measles Measles (C) is a viral respiratory system infection caused by the genus Morbillivirus. Infection presents with cough, coryza, conjunctivitis and/ or a discrete maculopapular rash. White spots on the buccal mucosa (Koplik spots) are pathognomonic for measles. Complications of measles infection may involve the respiratory (pneumonia and tracheitis) and neurological (febrile convulsions and encephalitis) systems. Subacute sclerosing panencephalitis (SSPE) may occur several years after the primary infection; infection persists in the central nervous system leading to loss of neurological function, dementia and eventually death.

109
Q

A Rubella B Syphilis C Measles D Hepatitis B E Mumps F Listeria monocytogenes G Cytomegalovirus H Haemophilus influenzae I HIV A 4-year-old boy presents to accident and emergency with a reduced level of consciousness, headache and neck stiffness. Analysis of the CSF reveals the presence of Gram-negative rods. The child’s mother reveals that his immunization record is not complete as they have only migrated from Ethiopia recently.

A

H Haemophilus influenzae Haemophilus influenzae (H) is a Gram-negative rod shaped bacterium that causes meningitis in children older than 3 months who have not been vaccinated. Other organisms that cause meningitis in older children include Streptococcus pneumoniae and Neisseria meningitidis. Diagnosis involves culture of the bacteria using chocolate agar, with subsequent Gram-stain and microscopy. Latex particle agglutination and PCR are more sensitive and specific investigative tests. The Haemophilus influenzae type B (Hib) vaccine has dramatically reduced Hib-related meningitis; the first dose is given when the child is 8 weeks old.

110
Q

A Rubella B Syphilis C Measles D Hepatitis B E Mumps F Listeria monocytogenes G Cytomegalovirus H Haemophilus influenzae I HIV An 8-month old girl is seen by a paediatrician due to concerns about developmental delay. On examination cataracts are noted in both eyes. Echocardiography reveals a patent ductus arteriosus.

A

A Rubella Rubella (German measles; A) is a viral infection which can be congenital or acquired. Congenital rubella syndrome (CRS) occurs in a developing fetus if the mother has contracted rubella in her first trimester. CRS is characterized by sensorineural deafness, eye abnormalities (cataracts, glaucoma, retinopathy) and congenital heart disease (patent ductus arteriosus). Other associations include microcephaly and developmental delay. Acquired rubella is transmitted via the respiratory route. Characteristically, a rash appears on the face which spreads to the trunk and disappears after a few days.